Vous êtes sur la page 1sur 48

TORTS OUTLINE - BOND

Purpose of Torts
1. Provide a peaceful means for adjusting the rights of parties who might otherwise "take the law into their own hands" 2. Deter wrongful conduct 3. Encourage socially responsible behavior 4. Restore injured parties to their original condition, insofar as the law can do this, by compensating them for their injury 5. Vindicate individual rights of redress

Exam approach:
First, review the fact pattern to spot each individual tort that has, or may have been, committed. Then, for each tort you have identified: 1. Prima facie case: Say whether a prima facie case for that tort has been made. 2. Defenses: Analyze what defenses and justifications, if any, D may be able to raise. 3. Damages: Finally, discuss what damages may be applicable, if the tort has been committed and there are no defenses. Pay special attention to: (1) punitive damages; (2) damages for emotional distress; (3) damages for loss of companionship of another person; (4) damages for unlikely and far-reaching consequences; and (5) damages for economic loss where there has been no personal injury or property damage.

3 Types of Torts
1) Intentional Torts: behavior that is intentional in some way, causes injury, and is deemed tortious 2) Negligent Torts: behavior that unreasonably risks personal/property injury to another. Here, the defendant has not intended to bring about a certain result, but has merely behaved carelessly. There are no individually-named torts in this category, merely the general concept of "negligence." 3) Strict Liability Torts: Behavior that is tortious because it causes unlawful personal/property damage to another, regardless of fault, reasonableness, etc. Here, the defendant is held liable even though he did not intend to bring about the undesirable result, and even though he behaved with utmost carefulness. There are two main individually-named torts that apply strict liability: a. Conducting of abnormally dangerous activities (e.g., blasting); and b. The selling of a defective product which causes personal injury or property damage.

Intentional Torts
A. Meaning of intent: There is no general meaning of "intent" when discussing intentional torts. For each individual intentional tort, you have to memorize a different definition of "intent." All that the intentional torts have in common is that D must have intended to bring about some sort of physical or mental effect upon another person. [6 - 7] 1. Act w Purpose or Desire desire to bring about certain consequences even if No intent to harm: The intentional torts generally are not defined in such a way as to require D to have intended to harm the plaintiff. (Example: D points a water gun at P, making it seem like a robbery, when in fact it is a practical joke. If D has intended to put P in fear of imminent harmful bodily contact, the "intent" for assault is present, even though D intended no "harm" to P.) 2. Substantial certainty: If D knows with substantial certainty that a particular effect will occur as a result of her action, she is deemed to have intended that result. (Example: D pulls a chair out from under P as she is sitting down. If D knew with "substantial certainty" that P would hit the ground, D meets the intent requirement for battery, even if he did not desire that she do so. [Garratt v. Dailey]) a. High likelihood: But if it is merely "highly likely," not "substantially certain," that the bad consequences will occur, then the act is not an intentional tort. "Recklessness" by D is not enough. 3. Act distinguished from consequences: Distinguish Ds act from the consequences of that act. The act must be intentional or substantially certain, but the consequences need not be. (Example: D intends to tap P lightly on the chin to annoy him. If P has a "glass jaw," which is broken by the light blow, D has still "intended" to cause the contact, and the intentional tort of battery has taken place, even though the consequences broken jaw were not intended.)

a. Restatement (2nd) 2: An act is an external manifestation of the actors will and does not include any of its results, even its most direct, immediate, and intended. b. To illustrate, comments clarify: when actor points a pistol at another person & pulls the trigger, the act is the pulling of the trigger. The consequence of that act is the impingement of the bullet upon the others person. (Example: In a forest area, Ken deliberately pulls the trigger of a rifle. He hopes to hit a wild deer, and he is unaware that any person is in the vicinity. The gun discharges. In fact, Nancy is nearby and is struck by the bullet. Ken has intentionally shot his gun. Yet he has not intentionally caused the harm to Nancy; he did not act with the purpose to produce that harm nor did he know that the harm was substantially certain to occur. 1. Restatement 3d: intent refers to consequences (Ken, the hunter).

(A) (B) (C) (D) (E)

Intent is subjective (will look at what D desired or knew) & but look to objective, circumstantial indicators of intent. (Garratt v. Dailey) Continuum of intentionality, including intent and negligence (the distinction between intent and negligence boils down to a matter of degree.) (Spivey v. Battaglia). Mistake does not negate intent. (Ranson v. Kitner) (no need malicious motive) Mental illness does not negate intent. (McGuire v. Almy). Kids (but not infants) are capable of intent. (Garratt v. Dailey).

Transferred Intent
1. Between People: Talmage v Smith 2. Between 5 original trespass torts (assault, battery, false imprisonment, trespass to land, trespass to chattels) 3. DNA to IIED

Battery:
A. Definition: Battery is the intentional infliction of a harmful or offensive bodily contact. (Example: A intentionally punches B in the nose. A has committed battery.) In a rude, insolent, harmful or angry manner 1. Any touching, however slight, MAY constitute an assault and battery 1. We all have an intuitive sense whether a touch is harmful. Holmes observed: even a dog know the difference between being tripped over and being kicked. (Wallace v. Rosen) B. Intent: It is not necessary that D desires to physically harm P. D has the necessary intent for battery if it is the case either that: (1) D intended to cause a harmful or offensive bodily contact; or (2) D intended to cause an imminent apprehension on Ps part of a harmful or offensive bodily contact. Example 1: D shoots at P, intending to hit him with the bullet. D has the necessary intent for battery. Example 2: D shoots at P, intending to miss P, but also intending to make P think that P would be hit. D has the intent needed for battery (i.e., the "intent to commit an assault" suffices as the intent for battery). No need for harmful intent/malicious to actually hurt somebody if there is an intent to bring about consequences C. Harmful or offensive contact: If the contact is "harmful" i.e., it causes pain or bodily damage this qualifies. But battery also covers contacts which are merely "offensive," i.e., damaging to a "reasonable sense of dignity." Example: D spits on P. Even if P is not "harmed" in the sense of being caused physical pain or physical injury, a battery has occurred because a person of average sensitivity in Ps position would have her dignity offended. D. P need not be aware: It is not necessary that P have actual awareness of the contact at the time it occurs. (Example: D kisses P while she is asleep. D has committed a battery.) E. P's body (or an extension of the body is an offensive invasion of a person - just as actual contact w the body Fisher v. Carousal expanded the notion of Body) F. Judge offensiveness on RP standard (Wallace v. Rosen) 1. Prosser n Keetons Crowded World: A certain amount of Personal contact is inevitable and must be accepted (Wallace v. Rosen fire alarm)

2. unless D knows P is particularly vulnerable 2. Will look to relationship between the parties 3. (Vosburg v. Putney) schoolboy playfully kicked another on the shin. Did not intend harm but did intend to kick him. Liable for battery even though the injury could not have been foreseen. 4. Do-gooders can be liable. (P fell at roller rink broken arm over the protests of P D manipulated the arm in an attempt to set it) 5. Damages: Punitive is especially egregious otherwise - $ to put them back where they were 6. AIDS case battery action alleged - consent to sex does not bar action.

Assault
POLICY REASONS - Why should the law provide any remedy if there is no bodily harm? Prevent others from acting out in violence as they might w assault. deterrence One of the goals of the tort system is to cut down on vigilante justice You might not recover from a battery if you have to wait till you get shot/struck w the hatchet Concerned not about IF they can actually to the harm, but if it APPEARS that they can A. Definition: Assault is the intentional causing of an apprehension of harmful or offensive contact. an unlawful attempt to commit a battery a. Example: D, a bill collector, threatens to punch P in the face if P does not pay a bill immediately. Since D has intended to put P in imminent apprehension of a harmful bodily contact, this is assault, whether D intends to in fact hit P or not. b. Apprehension (less than fear) of battery (the victim must have an APPREHENSION of contact, it is not necessary that the D have the actual ABILITY to carry out the threatened contract- Western Union v. Hill) c. The apprehension is the harm. Its sufficient (I de S) B. Intent: There are two different intents, either of which will suffice for assault: 1. Intent to create apprehension: First, D intends to put P in imminent apprehension of the harmful or offensive contact, even if D does not intend to follow through (e.g., D threatens to shoot P, but does not intend to actually shoot P); or 2. Intent to make contact: Alternatively, D intends to in fact cause a harmful or offensive bodily contact. (Example: D shoots a gun at P, trying to hit him. D hopes P wont see him, but P does. P is frightened, but the shot misses. This is assault.) 3. Summary: So D has the requisite intent for assault if D either "intends to commit an assault" or "intends to commit a battery." C. No hostility: It is not necessary that D bear malice towards P, or intend to harm her. (Example: D as a practical joke points a toy pistol at P, hoping that P will falsely think that P is about to be shot. D has one of the two alternative intents required for assault the intent to put P in imminent apprehension of a harmful or offensive contact so the fact that D does not desire to "harm" P is irrelevant.) D. "Words alone" rule: Ordinarily, words alone are not sufficient, by themselves, to give rise to an assault. Normally there must be some overt act a physical act or gesture by D before P can claim to have been assaulted. (Example: During an argument, D says to P "Im gonna hit you in the face." This is probably not an assault, if D does not make any gesture like forming a fist or stepping towards P.) 1. Special circumstances: However, the surrounding circumstances, or Ds past acts, may occasionally make it reasonable for P to interpret Ds words alone as creating the required apprehension of imminent contact. OR exceptions: (your money or your life) C. E. Imminence: It must appear to P that the harm being threatened is imminent, and that D has the present ability to carry out the threat. (Example: D threatens to shoot P, and leaves the room for the stated purpose of getting his revolver. D has not committed an assault on P.) (a well-founded fear of an imminent battery Western Union v. Hill) a. The perpetrator must possess the apparent present ability to effectuate the attempt. b. P must think D is have been able to carry out the battery - DS perception must be reasonable i. RP standard c. Q for the jury if D could have carried out battery - Context is important F. P unaware of danger: P must be aware of the threatened contact.

G. Threat to third persons: P must have an apprehension that she herself will be subjected to a bodily contact. She may not recover for her apprehension that someone else will be so touched. (Example: P sees D raise a pistol at Ps husband. D shoots and misses. P cannot recover for assault, because she did not fear a contact with her own body.) H. Conditional threat: Where D threatens the harm only if P does not obey Ds demands, the existence of an assault depends on whether D had the legal right to compel P to perform the act in question. (Example: P, a burglar, breaks into Ds house. D says, "If you dont get out, Ill throw you out." There is no assault on P, since D has the legal right to force P to leave.)

False Imprisonment
A. Definition: False imprisonment is defined as the intentional infliction of a confinement. Example: D wants to have sex with P, and locks her in his bedroom for two hours hoping that P will agree. She does not, and D lets her go. This is false imprisonment, because D has intentionally confined P for a substantial time. a. direct restraint of one person of the physical liberty of another w/out adequate legal justification Big Town Nursing Home v. Newman B. Intent: P must show that D either intended to confine him, or at least that D knew with substantial certainty that P would be confined by Ds actions. The tort of false imprisonment cannot be committed merely by negligent or reckless acts. (Example: D, a shopkeeper, negligently locks the store while P, a customer, is in the bathroom. This is not false imprisonment, since D did not intend to confine P.) C. "Confinement": The idea of confinement is that P is held within certain limits, not that she is prevented from entering certain places. (Example: D refuses to allow P to return to her own home. This is not false imprisonment P can go anywhere else, so she has not been "confined.") a. P must show he was restrained unlawfully and against his will (Hardy v. LaBell's Distributing CO. - Hardy admitted that even though she felt compelled to stay in the office, she wanted to stay anyway. Therefore, the court argues, it is reasonable for a jury to find that the plaintiff was not held against her will.)

D. Means used: The imprisonment may be carried out by direct physical means, but also by threats or by the assertion of legal authority. 1. Threats: Thus if D threatens to use force if P tries to escape, the requisite confinement exists. 2. Assertion of legal authority: Also, confinement may be caused by Ds assertion that he has legal authority to confine P this is true even if D does not in fact have the legal authority, so long as P reasonably believed that D does, or is in doubt about whether D does. (Example: Storekeeper suspects P of shoplifting, and says, "I hereby make a citizens arrest of you." Putting aside whether Storekeeper has a privilege to act this way, Storekeeper has "confined" P, if a reasonable person in Ps position would think that Storekeeper had the authority to make such an arrest, even if under local law Storekeeper did not have that authority.) E. P must know of confinement: P must either be aware of the confinement, or must suffer some actual harm. (Example: P is locked in her hotel room by D, but P is asleep for the entire three-hour period, and learns only later that the door was locked. This is probably not false imprisonment.) 1. (Parvi v. City of Kingston) drunk but still aware 2. OR if unaware of it BUT harmed by it 1. Baby FI in room w out food dehydrated F. Analogy w key and lock. (If I am unlawfully locked in a room without access to a key, then I have been falsely imprisoned. Likewise, if I am kept on a yacht without access to a rowboat, I have been falsely imprisoned. Putting someone on a yacht, same as locking the door - Whittaker v. Sandford) a. Can there be FI in moving automobile -yes b. City/state = yes c. Country = no (Taiwan) d. FI does not need to be small space

2. When P makes escape, P bears some of the risk/burden in assessing that its reasonable. Student who moves out of moving bus = unreasonable 3. If obvious escape - not FI a. If 1 exit is locked, but another one is open = no 4. If its a joke? = Yes 5. P would not be required to make escape thru a sewer 6. Shopkeeper's privilege: can reasonably hold someone to determine if you've actually shoplifted (POLICY) 7. What if stuck on tarmac for 11hrs. W no food/water/toilet doesnt work? Law on airlines side. You consent to be on an airplane. Most likely would settle 8. Transferred intent applies. Mistake is no excuse. 9. Good faith belief that confinement is beneficial is no excuse. a. Courts want to stay away from determining a cult/religion 10. False Arrest can be FI if arrest them for something not against the law a. unless probable cause or a warrant produced, can't arrest. UNLAWFUL ARREST b. The court reasons that the officer did not have proper legal authority because there he evidence he did not arrest Groves for the dog leash violation, but rather for failing to produce her drivers license. (Enright v. Groves).

IIED
The old rules state that the law will not protect individuals from invasions of their emotional and mental well-being. However, the court cites a change in the law reflected in the Restatement of Torts that says there may be an action for infliction of severe emotional distress (State Rubbish Collectors Association v. Siliznoff). A. Definition: This tort is the intentional or reckless infliction, by extreme and outrageous conduct, of severe emotional or mental distress, even in the absence of physical harm (words are enough). i. Example: D threatens that if P, a garbage collector, does not pay over part of his garbage collection proceeds to D and his henchmen, D will severely beat P. Since Ds conduct is extreme and outrageous, and since he has intended to cause P distress (which he has succeeded in doing), D is liable for infliction of mental distress. [State Rubbish Collectors Assoc. v. Siliznoff] B. Intent: "Intent" for this tort is a bit broader than for others. There are three possible types of culpability by D: (1) D desires to cause P emotional distress; (2) D knows with substantial certainty that P will suffer emotional distress; and (3) D recklessly disregards the high probability that emotional distress will occur. (Example: D commits suicide by slitting his throat in Ps kitchen. D, or his estate, is liable for intentional infliction of mental distress because although P did not desire to cause distress to P, or even know that distress was substantially certain, he recklessly disregarded the high risk that distress would occur.) i. Transferred intent: (D dont know she was watching when they beat up her dad not liable Taylor v. Vallelunga) Intent is important, bc thats the tort: INTENTIONAL IED ii. Immediate family present: The main exception is that the transferred intent doctrine is applied if: (1) D directs his conduct to a member of Ps immediate family; (2) P is present; and (3) Ps presence is known to D. (Example: While P is present, and known to D to be present, D beats up Ps father. If P suffers severe emotional distress, a court will probably allow her to recover from D, even though Ds conduct was directed at the father, not P.) C. "Extreme and outrageous": P must show that Ds conduct was extreme and outrageous. Ds conduct has to be "beyond all possible bounds of decency." High Threshold Example: D, as a practical joke, tells P that her husband has been badly injured in an accident, and is lying in the hospital with broken legs. This conduct is sufficiently outrageous to qualify. i. ("Liability only for conduct exceeding all bounds which could be tolerated by society Slocum v. Food Fair Stores of Florida). D. Actual severe distress: P must suffer severe emotional distress. P must show at least that her distress was severe enough that she sought medical aid. Most cases do not require P to show that the distress resulted in bodily harm. a. P must be affected in SEVERE way (As a result of the threats, P got physically ill and missed several days of work State Rubbish Collectors Association v. Siliznoff).

b. (stutterer - humiliation suffered was not, as a matter of law, so intense as to constitute the 'severe' emotional distress required to recover for the tort of intentional infliction of emotional distress. - Harris v. Jones). Crying is not enough c. POLICY: Courts have gate-keeping function. Not Efficient to take any case of hurt feeling. Crowded world sometimes feelings will get hurt D. There must be a causal connection between the wrongful conduct and the emotional distress (Harris v. Jones he already stuttered before IIED) E. Context is important: If KNOW of particular susceptibility/vulnerability and exploit it liable a. Pregnant women are presumably especially physically sensitive i. If not showing may not be liable knowledge is required b. courts seem to recognize that people are particularly sensitive about religion and race c. kids/rape victim are more vulnerable F. Common Carriers may be held to a higher standard. a. Buses/public transport. Not general businesses like in Slocum.

Trespass to Land
A. Definition: As generally used, "trespass" occurs when either: (1) D intentionally enters Ps land, without permission; (2) D remains on Ps land without the right to be there, even if she entered rightfully; or (3) D puts an object on (or refuses to remove an object from) Ps land without permission EXAMPLE: (Rogers v. Board of Road Comrs for Kent County D failed to remove a post as agreed, and P ran into it on his mower and died. He trespass by continued presence may lie if consent of some objects presence has been terminated) The act, crossing the border - Merely 'shooting the gun' is enough B. Intent: The term "trespass" today refers only to intentional interference with Ps interest in property. There is no strict liability. (Example: D, a pilot, loses control of the aircraft, and the aircraft lands on Ps property. This is not trespass to land.) 1. Negligence: If D negligently enters Ps land, this is generally treated as the tort of negligence, not trespass. What if trip into the land? Not liable - Got to be some sort of volition. (See Cohen v. petty when guy faints while driving and is not liable) C. Particles and gasses: If D knowingly causes objects, including particles or gases, to enter Ps property, most courts consider this trespass. (Example: Ds factory spews pollutants onto Ps land. This is a trespass) D. Air space: It can be a trespass for a plane to fly over Ps property. However, today most courts find liability only if: (1) the plane enters into the immediate reaches of the airspace (below federally-prescribed minimum flight altitudes); and (2) the flight substantially interferes with Ps use and enjoyment of his land (e.g., by causing undue noise, vibrations, pollution). (Herrin v. Sutherland disturbing peaceful enjoyment of land is a trespass) 3. NO need to show damages 4. Just have to intend to do the act of walking - dont have to attend consequences of trespassing.

Trespass to Chattels
A. Definition: "Trespass to chattels" is defined as any intentional interference with a persons use or possession of a chattel. D only has to pay damages, not the full value of the property (as in conversion, below). 1. Loss of possession: If P loses possession of the chattel for any time, recovery is allowed even if the chattel is returned unharmed. (Example: D takes Ps car for a five-minute "joy ride," and returns it unharmed. D has committed trespass to chattels.) Example: CompuServe Dont need to show actual dispossession of the chattel to find for P B. P has to SHOW Damages Example: Glidden v. Szybiak The court finds no evidence in the record to show that Elaine injured the dog. Therefore, the court concludes Elaine did not commit a tort at the time of her injury. HARMLESS MEDDLING IS NOT TRESSPASS TO CHATTLES

Conversion
A. Definition: Conversion is an intentional interference with Ps possession or ownership of property that is so substantial that D should be required to pay the propertys full value. Example: D steals Ps car, then seriously (though not irreparably) damages it in a collision. D is liable for conversion, and will be required to pay P the full value of the car (though D gets to keep the car). You have to replace it - whole amount of damage i. Case: Pearson v. Dodd - The court says that the documents themselves couldnt have been converted, because they were returned in useable condition. The court then considers whether the documents could be considered intellectual property that could be compromised by being merely seen and then returned. The court decides that the records that were copied, consisting of letters from constituents and office documents, are not intellectual property. B. Intent: Conversion is an intentional tort, but all that is required is that D have intended to take possession of the property. Mistake as to ownership will generally not be a defense. (Example: D buys an old painting from an art dealer, and reasonably believes that the art dealer has good title. In fact, the painting was stolen from P years before. D keeps the painting in his house for 10 years. D is liable for conversion, notwithstanding his honest mistake about title.) C. Distinguished from trespass to chattels: Courts consider several factors in determining whether Ds interference with Ps possessory rights is severe enough to be conversion, or just trespass to chattels. Factors include: (1) duration of Ds dominion over the property; (2) Ds good or bad faith; (3) the harm done to the property; and (4) the inconvenience caused to P. D. Different ways to commit: There are different ways in which conversion may be committed: 1. Acquiring possession: D takes possession of the property from P. (Stealing it - Damaging it - Using it) 2. Buying it from a thief - Bona fide purchaser: Most courts hold that a bona fide purchaser of stolen goods is a converter, even if there is no way he could have known that they were stolen. Good Faith does not matter 3. Transfer to third person: D can also commit conversion by transferring a chattel to one who is not entitled to it. (Example: -Delivering it to the wrong person D, a messenger service, delivers a package to the wrong person, X. X absconds with the goods. D has committed conversion, even though D did not end up with possession of the goods.) 4. Withholding good: D may commit conversion by refusing to return goods to their owner, if the refusal lasts for a substantial time. (Example: D, a parking garage, refuses to give P back her car for a day.) 5. Destruction: Conversion may occur if D destroys the goods, or fundamentally alters them. 6. Selling it to somebody else E. Forced sale: If P is successful with her tort suit, a forced sale occurs: D is required to pay the full value of the goods (not just the amount of the use or damage, as in trespass to chattels), but gets to keep the goods. The Restatement tries to spell out what makes a particular interference with chattels into a conversion: One way to smell a conversion is the serious interference test. How serious does the interference have to be? It must be serious enough that damages for the full market price of the chattel should be awarded. That seems like a suspiciously circular definition. Some other criteria for identifying a conversion include how long the actor had control of the chattel, how long the interference lasted, the actors good faith, how much harm was done to the chattel, and how inconvenience was caused to the other person (B) Necessity of Demand; Return of Chattel In a few jurisdictions, the owner of a chattel must demand their property back from the possessor and be rejected. In most jurisdictions, conversion happens as soon as someone unlawfully takes possession of the chattel. If the possessor gives the chattel back to the owner, it doesnt bar an action for conversion, but the damages will be mitigated. The owner doesnt have to accept the return of the goods, except that in some jurisdictions, if the conversion was innocent and the chattel wasnt damaged, the owner must take back the goods and wont have an action for conversion.

Damages 1. If something gets converted from yours to theirs, youre due the full value of the property converted. How this is calculated can vary, but usually its done by market price. Its kind of analogous to the rule in contracts that the owner ought to mitigate damages by buying a suitable replacement soon after the conversion. a. Though the property may have special value to the owner, only market price may be recovered unless the defendants conduct is outrageous, in which case the plaintiff may be able to get damages for emotional harm. Hypothetical: Bush v. Simpson (D) What May Be Converted Conversion originated from trover, which applied to things that can be lost and found. However, conversion has now been extended to apply to intangibles such as stock holdings. (E) Who May Maintain the Action Anyone who has the chattel when its converted can sue for conversion. One converter can even recover from another. Usually, the plaintiff can show some right to the converted good.

(C)

DEFENSES TO Intentional Torts I. CONSENT


A. Express consent: If P expressly consents to an intentional interference with his person or property, D will not be liable for that interference. (Example: P says to D, "Go ahead, hit me in the stomach Ill show you how strong I am." If D does so, Ps consent prevents P from suing for battery.) B. Implied consent: Existence of consent may also be implied from Ps conduct, from custom, or from the circumstances. 1. Objective manifestation: It is the objective manifestations by P that count if it reasonably seemed to one in Ds position that P consented, consent exists regardless of Ps subjective state of mind. (Example: D offers to vaccinate all passengers on their ship. P holds up her arm and receives the vaccination. Since it reasonably appeared by her ACTIONS to D that P consented, there will be consent regardless of Ps actual state of mind. *OBrien v. Cunard]) 2. Consent may be inferred from: A. Custom ex. Of fishing pond without No trespass / Shoulder tap/Jostling B. Relationship or Prior dealings between parties man kisses his finance v. stranger C. Lack of capacity: Consent will be invalidated if P is incapable of giving that consent, because she is a child, intoxicated, unconscious, etc. 1. Consent as a matter of law: But even if P is incapable of truly giving consent, consent will be implied "as a matter of law" if these factors exist: (1) P is unable to give consent; (2) immediate action is necessary to save Ps life or health; (3) there is no indication that P would not consent if able; and (4) a reasonable person would consent in the circumstances. (Example: P is brought unconscious to the emergency room of D, a hospital. D can perform emergency surgery without Ps actual consent consent will be implied as a matter of law. Therefore, P cannot sue for battery.) 2. If the patient, while competent, has reliably expressed her opposition to a particular medical procedure, her wishes are not to be overridden when she falls unconscious and death is imminent. Rodriguez v. Pino, 3. Minor Consent of the parent is necessary for any medical procedure except in an emergency. Timing and proximity of parents is relevant D. Exceeding scope: Even if P does consent to an invasion of her interests, D will not be privileged if he goes substantially beyond the scope of that consent. Example: P visits D, a doctor, and consents to an operation on her right ear. While P is under anesthetic, D decides that Ps left ear needs an operation as well, and does it. Ps consent does not block an action for battery for the left-ear operation, since the operation went beyond the scope of Ps consent. *Mohr v. Williams] In MEDICAL context - must be expressed (Informed Consent must know of surgery risks is now treated as negligence but when Dr. exceeds bounds of consent thats still Battery)

Example 2: Voluntarily participating in professional sports, where it is inherently dangerous, does not allow one player to strike another player out of frustration and anger. Contrary to the rules the defendant struck the PL. The DFs liability is one of an evidentiary finding as to whether PLs rights had been violated. Hackbart v. Cincinnati Bengals 1. Emergency: However, in the surgery case, an emergency may justify extending the surgery beyond that consented to. E. Consent to criminal acts: Where Ds act against P is a criminal act, courts are split. The majority rule is that Ps consent is ineffective if the act consented to is a crime. (Example: P and D agree to fight with each other. In most states, each may recover from the other, on the theory that consent to a crime such as breach of peace is ineffective.) Minority & Restatement: Consent to criminal conduct is valid unless statute protecting a class of people regardless of their consent (i.e. statutory rape) G. Under Duress - not valid H. Consent obtained through fraud/misrepresentation is not valid (De May v. Roberts - No person has the right to intrude upon a persons privacy or person, unless invited, (without deceit), or b/c some real or pressing necessity exists. Dr. guilty of deceit, which negates the consent given for the examination and treatment)

II. Self Defense


A. Privilege generally: A person is entitled to use reasonable force to prevent any threatened harmful or offensive bodily contact, and any threatened confinement or imprisonment. B. Apparent necessity: Self-defense may be used not only where there is a real threat of harm, but also If D mistakenly but reasonably believes that self-defense is necessary, privilege exists. 1. Provocation: Verbal insults do not justify SD. A threat must be imminent, but one does not have to wait for the blow to fall before acting. C. Only for protection: The defense of self-defense applies only where D uses the force needed to protect himself against harm. 1. Retaliation: Thus D may not use any degree of force in retaliation for a tort already committed. (Example: P hits D with a snowball. Ten minutes later, D hits P with a snowball, in retaliation. D has committed battery on P, because Ds act was not done in true self-defense.) 2. Imminence: D may not use force to avoid harm which is not imminent, unless it reasonably appears that there will not be a later chance to prevent the danger. (Example: P says to D, "I will beat you up tomorrow." D cannot beat P up today, to prevent tomorrows attack, unless it appears that there will be no way for D to defend tomorrow.) D. Degree of force: Only the degree of force necessary to prevent the threatened harm may be used. If D uses more force than necessary, he will be liable for damage caused by the excess. 1. Deadly force: Special rules limit the use of deadly force, i.e., force intended or likely to cause death or serious bodily injury. a. Danger must be serious: D may not use deadly force unless he himself is in danger of death or serious bodily harm. (Example: P attacks D with his fists, in a way that does not threaten D with serious bodily harm. Even if there is no other way for D to prevent the attack, D may not use his gun to shoot P, even if the shot is intended only to injure P D must submit to the attack rather than use deadly force.) E. Retreat: Courts are split on whether and when D has a "duty to retreat" (i.e., to run away or withdraw) if the threatened harm could be avoided this way. 1. Restatement view: The Second Restatement holds that: (1) D may use non-deadly force rather than retreating; but (2) D may not use deadly force in lieu of retreating, except if attacked in his dwelling by one who does not reside in the dwelling. (Example: If P attacks D on the street with a knife, under the Restatement D may use his fists rather than running away, but may not use a gun rather than running away if running away would avoid the danger. If the attack took place in Ds home, where P was not also a resident, then D could use the gun.) Most J require that ur defending against a serious felony.

III. D of others

A. General rule: A person may use reasonable force to defend another person against attack. The same rules apply as in self-defense: the defender may only use reasonable force, and may not use deadly force to repel a non-deadly attack. 1. Reasonable mistake: The courts are split on the effect of a reasonable mistake. Older courts hold that the intervener "steps into the shoes" of the person aided, and thus bears the risk of a mistake. But Rest.2d gives a "reasonable mistake" defense to the intervener.

IV. D of property
A. General rule: A person may generally use reasonable force to defend her property, both land and chattels. 1. Warning required first: The owner must first make a verbal demand that the intruder stop, unless it reasonably appears that violence or harm will occur immediately, or that the request to stop will be useless. B. Mistake: The effect of a reasonable mistake by D varies: 1. Mistake as to danger: If Ds mistake is about whether force is necessary, D is protected by a reasonable mistake. (Example: D uses non-deadly force to stop a burglar whom he reasonably believes to be armed. In fact, the burglar is not armed. D can rely on the defense of property.) 2. Privilege: But if the owners mistake is about whether the intruder has a right to be there, the owners use of force will not be privileged. (Example: D reasonably believes that P is a burglar. In fact, P is a friend who has entered Ds house to retrieve her purse, without wanting to bother D. Even non-deadly force by D will not be privileged.) C. Deadly force: The owner may use deadly force only where: (1) non-deadly force will not suffice; and (2) the owner reasonably believes that without deadly force, death or serious bodily harm will occur. (Example: D sees P trespassing in Ps backyard. D asks P to leave, but P refuses. Even if there is no way to make P leave except by shooting at him, D may not do so, since Ps conduct does not threaten D with death or serious bodily harm.) 1. Burglary: But a homeowner is generally allowed to use deadly force against a burglar, provided that she reasonably believes that nothing short of this force will safely keep the burglar out. D. Mechanical devices: An owner may use a mechanical device to protect her property only if she would be privileged to use a similar degree of force if she were present and acting herself. 1. Reasonable mistake: An owners right to use a dangerous mechanical device in a particular case will be measured by whether deadly force could have been used against that particular intruder. (Example: D uses a spring gun to protect his house while he is away. If the gun shoots an actual burglar, and state law would have allowed D to shoot the burglar if D was present, then D will not be liable for using the spring gun. But if a neighbor, postal carrier, or someone else not engaged in a crime happened to enter and was shot, D would not have a "reasonable mistake" defense since D could not have fired the gun at such a person directly, the spring gun may not be used either.) 2. Katko v. Briney The value of human life and limb both to an individual and as a matter of public policy outweighs the potential damage to property. Thus, while a defendant may use reasonable force in defense of her property, he has no right to willfully and intentionally injure a trespasser in a manner that may result in loss of life or great bodily injury. The only exception is when the trespasser is committing a violent felony with the potential of endangering human life 1. General Rules for Spring Guns: Generally speaking, a possessor may not set a deadly trap or spring gun likely to cause grave bodily harm in order to protect property except to prevent entry into a dwelling place where there is a serious threat to the occupants or commission of a serious felony. But, lacunae in the cases create uncertainties about the precise rules. So, note that the rules vary according to jurisdiction. Deadly traps generally ok to protect in self-defense (trap to impede access to bedroom door at night is probably ok if significant threat but dynamite at the front door in daytime is not still has to reasonable even in a dwelling) Deadly traps Not ok to protect property interests except: Protection of dwelling against felonious entry (if trespasser is committing a felony of violence or one punishable by death) Some states have extended this privilege to other buildings. But D uses deadly traps at his risk: if spring gun injures child or other non-felonious entry, no privilege will protect D. Ex. Mere trespassers do not trigger privilege to use deadly traps.

Note that criminal statutes amend these rules in many jurisdictions. Warnings Restatement does not require a notice, but some courts have implied that the spring gun might be justified only if a warning is posted.

V. Recovery of Property/Chattels
A. Generally: A property owner has the general right to use reasonable force to regain possession of chattels taken from her by someone else. 1. Fresh pursuit: The privilege exists only if the property owner is in "fresh pursuit" to recover his property. That is, the owner must act without unreasonable delay. (Example: A learns that B has stolen a stereo and is in possession of it. A may use reasonable force to reclaim the stereo if he acts immediately, but not if he waits, say, a week between learning that D has the property and attempting to regain it.) 2. Reasonable force: The force used must be reasonable, and deadly force can never be used. 3. Wrongful taking: The privilege exists only if the property was taken wrongfully from the owner. If the owner parts willingly with possession, and an event then occurs which gives him the right to repossess, he generally will not be able to use force to regain it. (Example: O rents a TV to A. A refuses to return the set on time. O probably may not use reasonable force to enter As home to repossess the set, because As original possession was not wrongful.) 4. He has demanded return & been refused 5. Hodgeden v. Hubbard - Rule: Property owners have the right to recover chattel if it can be done without unnecessary violence to the person, or without breach of the peace. Furthermore, there is no privilege on the part of the wrongdoer to resist Actor may use reasonable force to recapture chattels if: B. Merchant: Where a merchant reasonably believes that a person is stealing his property, many courts give the merchant a privilege to temporarily detain the person for investigation. 1. Limited time: The detention must be limited to a short time, generally 10 or 15 minutes or less, just long enough to determine whether the person has really shoplifted or not. Then, the police must be called (the merchant may not purport to arrest the suspect himself). 2. Reasonable force short of bodily harm. A request to stay. 3. Bonkowski v. Arlan's Department Store There exists a privilege for merchants or shopkeepers to detain those whom they reasonably believe to have unlawfully taken chattels for a reasonable investigation and thereby avoid liability for false imprisonment there are two important issues with respect to the privilege: 1) Whether there was a reasonable belief that an item had been unlawfully taken; and 2) Whether the investigation undertaken in response to this belief was reasonable. Satisfaction of both elements will provide a defense to a claim of false imprisonment

VI. Public Necessity


A. General rule: Under the defense of "necessity," D has a privilege to harm the property interest of P where this is necessary in order to prevent great harm to third persons or to the defendant herself. B. Public necessity: If interference with the land or chattels of another is necessary to prevent a disaster to the community or to many people, the privilege is that of "public necessity." Here, no compensation has to be paid by the person doing the damage. (Example: Surocco v. Geary - Firefighters demolish Ds house, in which a fire has just barely started, because that is the best way to stop the fire from spreading much further. The firefighters, and the town employing them, probably do not have to pay, because they are protected by the privilege of public necessity.) It sucks/might not be fair but its not tort. C. Private necessity: If a person prevents injury to himself or his property, or to the person or property of a third person, this is protected by the privilege of "private necessity," if there is no less-damaging way of preventing the harm. (Example: Vincent v. Lake Erie A, while sailing, is caught in very rough seas. To save his life, he may moor at a dock owned by B, and will not be liable for trespass.) A persons life is worth more than property. Saving the crew = public necessity. Cant kick them out BC emergency 1. Actual damage: Where the privilege of private necessity exists, it will be a complete defense to a tort claim where P has suffered no actual substantial harm (as in the above example). But if actual damage occurs, P must

pay for the damage she has caused. (Example: On the facts of the above example, if As boat slammed into Bs dock and damaged it, A would have to pay.) 2. Owner may not resist: The main purpose of the doctrine of private necessity is to prevent the person whose property might be injured from defeating the exercise of the privilege. Example: P moors his ship at Ds dock, to avoid being shipwrecked by heavy seas. D, objecting to what he thinks is a trespass, unmoors the ship, causing the ship to be harmed and P to be injured. P may recover from D, because Ps mooring was privileged by private necessity and D, therefore, acted wrongfully. [Ploof v. Putnam,] Cant shove ppl back out into the storm. Then owner is liable for battery.

VII. Arrest
A. Common law rules: 1. Arrest with warrant: Where a police officer executes an arrest with an arrest warrant that appears to be correctly issued, he will not be liable even if it turns out that there was no probable cause or the procedures used to get the warrant were not proper. 2. Arrest without warrant: a. Felony or breach of peace in presence: A police officer may make a warrantless arrest for a felony or for a breach of the peace, if the offense is being committed or seems about to be committed in his presence. A citizen may do the same. b. Past felony: Once a felony has been committed, an officer may still make a warrantless arrest, provided that he reasonably believes that the felony has been committed, and also reasonably believes that he has the right criminal. A citizen may make an arrest only if a felony has in fact been committed (though the citizen is protected if she makes a reasonable mistake and arrests the wrong person). c. Misdemeanor: At common law, no warrantless arrest (either by an officer or by a citizen) may be made for a past misdemeanor not involving a breach of the peace. 3. Reasonable force: One making an arrest may not use more force than is reasonably necessary. a. Prevention: Where the arrest is made to prevent a felony which threatens human life or safety, even deadly force may be used, if there is no other way to prevent the crime. But where the felony does not involve such danger, deadly force may not be used. b. Apprehension after crime: If a crime has already been committed, the police may use deadly force only if the suspect poses a significant threat of death or serious physical injury to others. (Example: Officer spots Burglar escaping after his crime. Officer knows that Burglar is unarmed and unlikely to be violent. Officer may not shoot at Burglar to arrest him, even if there is no other way to make the arrest.)

VIII. Discipline
1. D against FI/Battery/Assault. 2. Parent/Child privilege can extend to those temporarily watching the child - but amount of force allowed is less a. Babysitter/aunt, etc. b. AGE/SEX/MOTIVE/REASONABLENESS/PERM HARM Factors help determine if disp. Was reasonable 2. Corporal punishment does not violate 8th amend cruel and unusual punishment or due process

IX. Justification
A. Generally: Even if Ds conduct does not fit within one of the narrower defenses, she may be entitled to the general defense of "justification," a catch-all term used where there are good reasons for exculpating D from what would otherwise be an intentional tort. Example: Sindle v. NY City Transit Authority - Bus case - A restraint or detention that is reasonable under the circumstances in time and manner and imposed for the purpose of preventing another from interfering with or damaging real or personal property is not unlawful. Jury can decide reasonableness of Ds actions.

NEGLIGENCE
Whenever we act we might hurt somebody else. Law says we want you to lower the chances of u hurting someone else. Law of neg makes us take risk reducing precautions to lower chance ull hurt someone else. How many risk reducing precautions? Must take RPP precautions acting under similar circumstances

I. COMPONENTS OF TORT OF NEGLIGENCE


A. Generally: The tort of "negligence" occurs when Ds conduct imposes an unreasonable risk upon another, which results in injury to that other. The negligent tortfeasors mental state is irrelevant. B. Prima facie case: The components of a negligence action are: Talk about each element in turn on EXAM. TALK about all 4 even though other elements may not be in dispute. The elements flow into each other to tell a continuous story 1. Duty: A legal duty requiring D to conduct himself according to a certain standard, so as to avoid unreasonable risk to others; STANDARD OF CARE RPP UNFORESEEABLE P (CARDOZO) 2. BREACH - Failure to conform: A failure by D to conform his conduct to this standard. (This element can be thought of as "carelessness.") APPLY STANDARD OF DUTY UNDER BREACH HAND FORMULA 3. CAUSATION BUT FOR AND Proximate cause: A sufficiently close causal link between Ds act of negligence and the harm suffered by P. This is "proximate cause." FORESEEABLE AND SCOPE OF RISK 4. DAMAGES - Actual damage: Actual damage suffered by P. (Compare this to most intentional torts, such as trespass, where P can recover nominal damages even without actual injury.) Cases: 1. Lubitz v. Wells - Wells left a golf club sitting in his yard. Wells left a golf club sitting in his yard REASONABLE CONDUCT W LOW PROP OF HARM = NO NEGL 2. Blyth v. Birmingham Waterworks Co. - OBJECTIVE RPP TEST APPLIES TO REASONABLY UNFORSEEN RISK - The defendants are negligent only if they fail to do what a reasonable person would have done or do something a reasonable person would not have done. - Negligence is situational: never had been a storm like this is "ordinary years". Not foreseeable. Rule: In order for a defendant to be held liable for negligence, s/he must have been able to foresee the injury to the plaintiff as a result of his/her failure to exercise due care Cohen v. Petty first time fainting not liable for accident caused when fainted while driving 3. Pipher v. Parsell - UNREASONABLE NOT TO FORESEE 2ND RISK - Fool me once, shame on you, Fool me twice, and I'm liable for negligence - In that 30 sec, driver should have done something - he was negligent 4. 5. Davison v. Snobhomish County - driving across a bridge and lost control. Their car skidded, broke through a railing, and fell to the ground. As a matter of public policy, we want roads to be built and maintained, and thus we cannot require safety measures that would make roads generally unaffordable.

NEGLIGENCE DUTY
I. UNREASONABLE RISK
A. Generally: P must show that Ds conduct imposed an unreasonable risk of harm on P (or on a class of persons of whom P is a member). 1. Not judged by results: It is not enough for P to show that Ds conduct resulted in a terrible injury. P must show that Ds conduct, viewed as of the time it occurred, without benefit of hindsight, imposed an unreasonable risk of harm. B. Balancing: In determining whether the risk of harm from Ds conduct was so great as to be "unreasonable," courts use a balancing test: "Where an act is one which a reasonable [person] would recognize as involving a risk of harm to another, the risk is unreasonable and the act is negligent if the risk is of such magnitude as to outweigh what the law regards as the utility of the act or of the particular manner in which it is done."

II. THE REASONABLE PERSON

A. Objective standard: The reasonableness of Ds conduct is viewed under an objective standard: EVERY PERSON HELD TO THE SAME STANDARD OF CARE. Would a "reasonable person of ordinary prudence," in Ds position, do as D did? D does not escape liability merely because she intended to behave carefully or thought she was behaving carefully. How many risk reducing precautions? Must take RPP precautions acting under similar circumstances. Same standard whether in pillow fight or high speed chase BC its all UNDER THE CIRCUMSTANCE (driving on a sunny day v. snowy day each case you drive like a RPP under the circumstances). THE STANDARD OF CARE REMAINS THE SAME Example: Delair v. Mcadoo - OBEJECTIVE test. - You know when you see the tire is worn through - its dangerous judge compares the worn out tire with a loaded gun - doesn't matter if you didnt know. A RP driver would know - WE EXPECT DRIVERS TO KNOW CAR BASICS, POLICY: the hazard it too great to permit cars in this condition on the highway 2 exceptions to RPP: 1. superior skill held to higher standard (similarly someone who knows the neighborhood held to higher standard when driving around a sharp corner. 2. Physical characteristics Physical and mental characteristics: The question is whether D behaved reasonably "under the circumstances." "The circumstances" generally include the physical characteristics of D himself. 1. Physical disability: Thus if D has a physical disability, the standard for negligence is what a reasonable person with that physical disability would have done. (Example: P is blind and is struck while crossing the street using a cane. If the issue is whether P was contributory negligent, the issue will be whether a blind person would have crossed the street in that manner. Roberts v. State of Louisiana - Establishes RULE Blind person held to standard of a reasonable prudent blind person, not a RPP) Watch for Red Herring blind ppl should still know not to set fire to kerosene. 2. Mental characteristics: The ordinary reasonable person is not deemed to have the particular mental characteristics of D. (Example: Vaughan v. Menlove - If D is more stupid, or more careless, than an ordinary person, this will not be a defense. EVERYONE HAS TO DO AS GOOD AS THE RPP) 3. Intoxication: Intoxication is no defense even if D is drunk, she is held to the standard of conduct of a reasonable sober person. 4. Children: special standard of care - A child is held to the level of conduct of a reasonable person of that age and experience and intelligence (A,E,I) acting under similar circumstances not that of an adult. Age 5-18 a. Adult activity: But where a child engages in a potentially dangerous activity normally pursued only by adults, she will be held to the standard of care that a reasonable adult doing that activity would exercise. (Example: Robinson v. Lindsay - If D operates a snowmobile, an activity that is potentially dangerous and normally pursued by adults, D must match the standard of care of a reasonable adult) hunting is debatable depending on state b. MOST J >5 incapable of negligence. More may be required of a child of superior intelligence. 2 yr olds dont owe duty of care to the rest of the world c. subjective diff SofCare for every kid. Pro-defendant. 6yr old, novice bike rider, stupid. Thats his standard. If drive bike like mentally defective 5yr old then liable. 5. Mental Illness: MENTALL HANDICAPPED/ An insane person is held to RPP standard even if that standard is unattainable for them. Insane is liable for negligence if there is a prior warning of the illness. (Exception: Breunig v. American Family Ins. Co. - ONSET IS SUDDEN N W OUT WARMING (Wisconsin specific) Policy: The general policy considerations for holding an insane person liable for his torts are: (1) when one of two innocent persons must suffer a loss it should be borne by the one who occasioned it; corrective justice (2) to induce those interested in the estate of the insane person to restrain and control him; incentivize caretaking (3) to stop false claims of insanity to avoid liability. Why? Created all kinds of problem w interpretation in crim context. 6. NOVICE beginning skier? Still held to RPP standard. C. Custom: Courts generally allow evidence as to custom for the purpose of showing presence or absence of reasonable care. However, this evidence is generally not conclusive. 1. Evidence by D but not dispositive: Thus where D shows that everyone else in the industry does things the way D did them, the jury is still free to conclude that the industry custom is unreasonably dangerous and thus negligent

(Example: D operates a tugboat without a radio; the fact that most tugboats in the industry do not yet have radios does not prevent the jury from holding that Ds lack of a radio was negligent. *The T.J. Hooper]) 1. Proof by plaintiff: Conversely, proof offered by P that others in Ds industry followed a certain precaution that D did not, will be suggestive but not conclusive evidence that D was negligent. (Trimarco v. Klein P introduced evidence that shows that it was a customary practice among landlords to install shatterproof shower doors.) CA. can also bring in Ps customs D. Emergencies: If D is confronted with an emergency, and is forced to act with little time for reflection, D must merely behave as a reasonable person would if confronted with the same emergency, not as a reasonable person would with plenty of time to think. (Example: D is a cab driver. A thief jumps in the cab, points a gun at Ds head, and tells him to drive fast. D, in a panic, mistakenly puts the car in reverse and injures P. The issue is whether a cab driver confronted with a gun-pointing thief would or might have behaved as D did, not whether a cab driver in ordinary circumstances would have behaved that way Cordas v. Peerless Transportation Co. Emergency SITUATION: d HAS NO OPPORTUNITY TO REFLECT, REASONABLE UNDER THE CIRCUMSTANCES When acting under the belief that his life is in danger and by abandoning the taxi he will save his life is not negligence.) Still have to act like a reasonable person would under an emergency If u brought on the emergency, cant use that as ur excuse In some cases you have a responsibility to anticipate an emergency. I.e. driving around a school yard and a kid darts out - duty to drive slow Sudden incapacity epileptic driver has to take their meds EXCEPTIONS TO THE RPP Standard of Care 1. Children see above 2. Professionals 3. Premises Liability Possessor of Land to protect entrants from dangerous conditions 4. Statutes 5. Duty to Act (no duty to rescue) 6. Pure Economic Loss 7. NIED 8. Unborn Children 9. Unforeseeable P

MALPRACTICE/Professionals Exception to RPP standard: added skill = held to higher standard


A. Superior ability or knowledge: If D has a higher degree of knowledge, skill or experience than the "reasonable person," D must use that higher level. (Example: D, because she is a local resident, knows that a stretch of highway is exceptionally curvy and thus dangerous. D drives at a rate of speed that one who did not know the terrain well would think was reasonable, and crashes, injuring her passenger, P. Even though Ds driving would not have represented carelessness if done by a reasonable person with ordinary knowledge of the road, D was responsible for using her special knowledge and is negligent for not doing so.) B. Malpractice generally: Professionals, including doctors, lawyers, accountants, engineers, etc., must act with the level of skill and learning commonly possessed by members of the profession in good standing. Average [reasonable] member of that profession providing the same service. In malpractice not a fictional RPP compared to real world colleague . demand that doctors/etc. conform. Evidence of custom is the standard of care ( need Expert) (Example: Heath v. Swift Winds, Inc. - The trial Ct. improperly introduced a subjective standard: an ordinary prudent pilot having the same training and experience as Fred Heath. This allows any jury to impose a different standard of care upon each individual. 1. Good results not guaranteed: The professional will not normally be held to guarantee that a successful result will occur, only that she will use the requisite minimum skill and competence. (Example: Hodges v. Carter- D mailed the process to the commissioner in following a custom which had prevailed in this State for over two decades - Professionals are not expected to know everything, just what the ordinary member of the profession does) Exercise Best Judgment - Discretionary matters - not liable for a mere error of judgment

Use Due Care - Mechanical matters. Ex: missed a statute of lim = liability. Crts. More likely to find fault here 2. Differing schools: If there are conflicting schools of thought within a profession, D must be judged by reference to the belief of the school he follows. (Example: An osteopath is judged by the standards of osteopathy, not the standards of medicine at large.) 3. Specialists: If D holds herself out as a specialist in a certain niche in her profession, she will be held to the minimum standard of that specialty. (Example: An M.D. who holds herself out as an ophthalmologist must perform to the level of the minimally competent ophthalmologist, not merely to the minimum level of the internist or general practitioner.) 4. Minimally qualified member: It is not enough for P to prove that D performed with less skill than the average member of the profession. D must be shown to have lacked the skill level of the minimally qualified member in good standing. a. Novice: One who is just beginning the practice of his special profession is held to the same level of competence as a member of the profession generally. (Example: A lawyer who has just passed the bar does not get the benefit of a lower standard he must perform at the level of minimally competent lawyers generally, not novices.) 5. Community standards: Traditionally, doctors and other professionals have been bound by the professional standards prevailing in the community in which they practice, not by a national standard. (Example: Traditionally, the "country doctor" need not perform with the skill commonly found in cities. Morrison v. MacNamara court abandons the locality rule) a. Change in rule: But this rule is on its way out, and many if not most courts would today apply a national standard. In "modern" courts, P may therefore use expert testimony from an expert who practices outside of Ds community. Now national Standard b. Boyce v. Brown - All dr. Kent could say he would do diff is an x-ray TEST: 1. To apply his knowledge n skill w ordinary and reasonable care 2. Must have done something or neglected to do something negligent 3. Must show what the standard med practice is 4. Show damages did result 5. Departure from the reasonable standard of care must be shown via expert testimony 6. That another dr. would have done it different is no sufficient to establish malpractice MALPRACTICE IS EXCEPTION - THE CUSTOM IS THE STANDARD OF CARE 6. Informed consent BOND SAYS REVIEW: In the case of a physician, part of the professional duty is to adequately disclose the risks of proposed treatment to the patient in advance. The rule requiring adequate disclosure is called the rule of "informed consent." The doctor must disclose to the patient all risks inherent in the proposed treatment which are sufficiently material that a reasonable patient would take them into account in deciding whether to undergo the treatment. Failure to get the patients adequate consent is deemed a form of malpractice and thus a form of negligence. (In some cases, usually older ones, failure to get informed consent transforms the treatment into battery.) (Scott v. Bradford - She signed a routine consent to surgery form prior to the hysterectomy - New rule: Full disclosure of all material risks incident to treatment must be made. A risk is material if it would be likely to affect a patients decision.) MEDICAL MALPRACTICE TEST - must prove: 1. D Physician failed to inform patient adequately of a MATERIAL risk before securing consent to the proposed treatment 2. If patent had been informed of the risks he would not have consented to the treatment 3. The adverse consequences that were not made known did in fact occur and he was injured as a result of submitting to the treatment The Defense against test. Dr. must prove 1. Patient knew of the risks OR 2. Full disclosure would be detrimental to patient's best interests OR 3. And emergency situation where patient needed prompt treatment was in no position to consent

Example 2: Moore v. Regents of the U of C - Dr/D used cells from his spleen for profit. A physician must tell the patient if he has personal interests that may affect his professional judgment. If he fails to do so, he may be liable for malpractice based on breach of informed consent Three principles: 1. An adult has the right to control his own body. 2. Consent is only effective if it is informed consent. 3. The doctor must tell you about everything that is material to your decision to give consent.

OWNERS AND OCCUPIERS OF LAND DUTY I. OUTSIDE THE PREMISES


A. Effect outside: There are special rules lowering a landowners standard of care. However, these rules do not apply to conduct by the landowner that has effects outside of his property. Therefore, the general "reasonable care" standard usually applies to such effects. 1. Natural hazards: However, if a hazardous condition exists naturally on the land, the property owner generally has no duty to remove it or guard against it, even if it poses an unreasonable danger to persons outside the property. But in an urban or other thickly-settled area, courts are less likely to apply this traditional rule. (Example: O allows a tree to grow in such a way that it may hit a tall truck passing on the roadway. Traditionally, O may not be held liable to the driver of the truck. But in an urban or suburban context, O might be liable.) Taylor v. Olsen - NATURAL CONDITIONS - this case is the exception to the rule of no duty to protect one outside the premises of natural conditions RULE: The standard of care a landowner owes in relation to roadside trees is reasonable care to prevent an unreasonable risk of harm 2. Artificial hazards: Where the hazardous condition is artificially created, the owner has a general duty to prevent an unreasonable risk of harm to persons outside the premises. EXAMPLE: Salevan v. Wilmington Park, Inc. ballpark RULE: Landowners who use their land in an artificial way must take reasonable precautions to protect the public traveling

II. TRESPASSERS Least amount of Duty 2 types:


#1. Undiscovered trespasser: zero duty of care cannot win a premises liability case
A. General rule: As a general rule, the landowner owes no duty to a trespasser to make her land safe, to warn of dangers on it, to avoid carrying on dangerous activities on it, or to protect the trespasser in any other way. (Example: P trespasses on D railroads track. His foot gets caught, and he is run over by a train. Even if the reason that P caught his foot was that D negligently maintained the roadbed, P cannot recover because D owed him no duty before discovering his presence. [Sheehan v. St. Paul Ry. Co.]) Rule: Railroad companies are under a constant and strict duty of care to individuals at railway crossings [bc can assume NOTICE at a public crossing], but are not bound to any act or service in anticipation of trespassers at other points along the track. Trespasser ASSUMES ALL RISK #2 2. Discovered trespassers: Once the owner has knowledge that a particular person is trespassing, the owner is then under a duty to exercise reasonable care for the trespassers safety.(Example: A railroads engineer must use reasonable care in stopping the train once he sees P trespassing on the tracks.) Discovered trespasser: come w out permission but owner knows about or should expect them to be there (discovered and anticipated trespassers) Constant trespass on a limited area: If the owner has reason to know that a limited portion of her land is frequently used by various trespassers, she must use reasonable care to make the premises safe or at least warn of dangers. This is the "constant trespass on a limited area" exception.(Example: If trespassers have worn a path across a railroad, the railroad must use reasonable care, such as whistles, when traversing that crossing.) 4 part test to trigger duty. Must meet all 4

1. 2. 3.

Artificial in Nature: built by man. No duty for natural conditions. Highly dangerous: would kill or badly injure someone. No duty if moderately dangerous. Condition must be concealed from trespasser hidden/secret. Do duty for open and obvious big trench on construction site 4. Prior knowledge by land owner ********Duty to protect against KNOWN, MANMADE, DEATH TRAPS on land

III. LICENSEES (social guest - people you Like)


A. Definition of licensee: A licensee is a person who has the owners consent to be on the property, but who does not have a business purpose (dont confer an economic benefit on possessor) for being there, or anything else entitling him to be on the land apart from the owners consent. *Social guests: The main class of persons who qualify as licensees are "social guests." (Example: Even if P is invited to Os house for dinner, P is a "licensee," not an "invitee.") Barmore v. Elmore - Plaintiff visited the home of discuss the business of the Masonic Lodge. During the visit, Thomas Jr., Thomas Sr.s 47 year old son, advanced towards Plaintiff with a steak knife. Defendants only duty was to warn him of hidden dangers unknown to Plaintiff of which Defendants had knowledge. *People who come to the door salesman/girlscout/guy who reads gas meter. You have an implied consent that she can walk up to the front door but shes not conferring an economic benefit on u. B. Duty to licensees: Duty to warn if its (1)CONCEALED and (2)KNOWN in advance BY POSESSOR (but unlike w trespasser dn matter if its nat/artificial or slightly/very dangerous). The owner does not owe a licensee any duty to inspect for unknown dangers. On the other hand, if the owner knows of a dangerous condition, she must warn the licensee of that danger. (Example: Rear steps leading from Os house to her back yard contain a rotten wood plank. If O knows of the rotten condition, she must warn P, a licensee, if P cannot reasonably be expected to spot the danger himself. But O need not inspect the steps to make sure they are safe, even if a reasonably careful owner would do so.) ********Duty to protect against KNOWN TRAPS on land

IV. INVITEES - highest level of duty - Duty to discover and warn about it or FIX it
B. Definition of "invitee": (1) Persons who are invited (have permission) by O onto the land to conduct business with O (ex: grocery store, any place of business) ; OR (2) Land is held open to the public at large (church, airport, museum) Case ex: Campbell v. Weathers - entered the business of D without purchasing any items, used the restroom injured when he stepped into an open trap door RULE: If an individual enters a store with the intention of then, or at some other time, doing business with that store, he is an invitee. Scope of invitation: If the visitors use of the premises goes beyond the business purpose or beyond the part of the premises held open to the public, that person will change from an invitee to a licensee. (Example: P visits Os store to buy cigarettes. O then allows P to use a private bathroom in the back of the store not held open to the public. Even though P was an invitee when he first came into the store, he becomes a licensee when he goes into the private bathroom. [Whelan v. Van Natta])RULE: A land possessor is only subject to the liability of another as an invitee for harm sustained while he is on the land within the scope of his invitation. A. Duty to invitee: 2 part test 1. reasonable inspection to find hidden dangers. The owner has a duty to inspect her premises for hidden dangers (CONCEALED). Knew about or should have known about. Doesnt mean they have to discover everything just what RPP would uncover. The RPP is careful but not obsessive. This is true even as to dangers that existed before O moved onto the premises. 2. Warning: The giving of a warning will often, but not always, suffices. If O should realize that a warning will not remove the danger, then the condition must actually be remedied. The owner must use reasonable care to take affirmative action to remedy a dangerous condition.

3. Control over third persons: Reasonable care by O may require that she exercise control over third persons on her premises. ****ALL REASONABLEY KNOWABLE TRAPS ON THE LAND

V. REJECTION OF CATEGORIES
A. Rejection generally: A minority of courts rejected the categories of trespasser, licensee and invitee. These courts now apply a general single "reasonable Prudent person" under the circumstances standard of liability. Rowland v. Christian *RULE: California instead applies ordinary principles of negligence in these cases. General Duty of Care = covered in beginning of neg. *Bc still using the under the circumstances standard. A RPP is going to protect customers MORE than you would a trespasser status of entrant still comes into play thru backdoor.

VI. Trespassing Children:


O must exercise reasonable care to prevent trespassing kids from getting hurt.

Attractive Nuisance Doctrine: how likely is it that children will trespass? the higher the likelihood the safer u should make ur property
The owner owes a duty of reasonable care to a trespassing child for an ARTIFICIAL CONDITION if: (1) the owner knows that the area is one where children are likely to trespass; (if live in middle of nowhere might be 0) (2) the owner has reason to know that the condition poses an unreasonable risk of serious injury or death to trespassing children; A. The condition Is one that children would be attracted to: kiddie Magnet swing set/pool (3) the injured child either does not discover the condition or does not realize the danger, due to his youth; (if child perceives the risk it DN apply - Impose some burden on a 5yr old but not a 2yr old) (4) the benefit to the owner of maintaining the condition in its dangerous form is slight weighed against the risk to the children; and (5) the owner fails to use reasonable care to eliminate the danger - fence in your pool (But dont have to fence in pond) Example: O knows that children often swim in a swimming pool on Os land. One part of the pool is unexpectedly deep. It would not cost very much for O to install fencing. P, a child trespasser, walks on the bottom of the pool, panics after suddenly reaching the deep part, and drowns. O is probably liable to P on these facts. a. Natural conditions: The court is less likely to find liability where the condition is a natural one than where it is artificial. b. No duty of inspection: The child trespass rules do not generally impose any duty of inspection upon O. Chicago B. & Q.R. Co. v. Krayenbuhl - A child playing on a railroad turntable - WHEN D KNOWS OR SHOULD KNOW KIDS WILL BE PLAYING THERE, D HAS A DUTY OF CARE TO PROTECT AGAINST HARM - Cost/Benefit analysis. Hand Formula Its about proportion and public good. A turntable is a dangerous contrivance, which facilitates railroading; the general benefits resulting from its use outweigh the occasional injuries inflicted by it; hence the public good demands its use. Regard must be had for 1. character and location of the premises 2. purpose for which it is used 3. probability of injury 4. precautions necessary to prevent the injury In all these cases there are 2 ways the owner can satisfy the duty Fix Condition Warn (barbri guy says a warning fully satisfies the duty) Be careful, carpet is slippery instead of putting a pad under carpet. or put up a sign

VI. LIABILITY OF LESSORS AND LESSEES


A. Lessee: A tenant is treated as if she were the owner all the rules of owner liability above apply to her. B. Lessors: In general, a lessor is not liable in tort once he transfers possession to the lessee. However, there are a number of exceptions to this general rule: 1. Known to lessor, unknown to lessee: The lessor will be liable to the lessee (and to the lessees invitees and licensees) for any dangers existing at the start of the lease, which the lessor knows or should know about, and which the lessee has no reason to know about. (This usually does not impose on the lessor a duty to inspect the premises at the start of the lease.) 2. Open to public: If the lessor has reason to believe that the lessee will hold the premises open to the public, the lessor has an affirmative duty to inspect the premises to find and repair dangers before the lease starts. 3. Common areas: Kline v. 1500 Mass. Ave Apartment Corp. P was assaulted and robbed in a common hallway of Defendants apartment. Defendant was on notice that assaults were occurring in the hallway RULE: Landlords have a duty to take steps to protect tenants from foreseeable criminal acts committed by third parties in common areas of landlords property. 4. Lessor contracts to repair: If the lessor contracts, as part of the lease, to keep the premises in good repair, most courts hold that the landlords breach of this covenant to repair gives a tort claim to anyone injured. However, P must show that D failed to use reasonable care in performing it is not enough to show that D breached the contract. 5. Negligent repairs: The landlord may incur liability even without a contractual repair obligation if she begins to make repairs, and either performs them unreasonably, or fails to finish them. This is clearly true where the landlord worsens the danger by performing the repair negligently. Courts are split about what happens where the landlord starts the repair, then abandons it, without worsening the danger. 6. General negligence standard: Courts that impose a general negligence standard on occupiers of land often impose a similar general requirement of due care upon lessors. (2) Conditions dangerous to persons outside the premises?? IMPLIED WARRANTY OF HABITABILITY: Pagelsdorf v. Safeco Ins. Co. of America Rule: Under Wisconsin law, a landlord owes to his tenant or anyone on the premises with the tenants consent a duty to exercise ordinary care. An implied warranty of habitability requires a lessor to deliver the premises to the lessee in a habitable condition. Generally, the only recourse for a landlords failure to comply with this warranty is a right to withhold rent until repairs are made.

VIOLATION OF STATUTE Special Duty Special Standard of Care


AUTOMOBILE GUEST STATUTES A. Generally: A minority of states still have "automobile guest statutes" on their books. These generally provide that an owner-driver is not liable for any injuries received by his non-paying passenger, unless the driver was grossly negligent or reckless. RULES OF LAW pg. 208 Black n white (BRIGHT LINE) v flexible standard. Pokorah v. Wabash. Benefits of rules v. flexible standard. BC neg cases are so fact specific bright line rules are impractical create consistency and be more efficient but makes the application of special circumstances difficult Pokora v. Wabash Ry. Co Brief Fact Summary. Plaintiff was killed while attempting to cross Wabash Ry. Co.s (Defendants) four railroad tracks. Plaintiff did not get out of his vehicle to obtain a better view as required by the opinion in Baltimore & Ohio R.R. v. Goodman. Synopsis of Rule of Law. To get out of a vehicle is uncommon precaution, as everyday experience informs us. The actions of a plaintiff depend on the situation and the circumstances, and it is up to the jury to decide whether a particular course of action was reasonable

VIOLATION OF STATUTE

Ordinary negligence case, the jury has a great deal of discretion, jury looks at Ds behavior whether reasonable/not but can go either way. Therefore when use a crim/statutory law as the standard of care makes it more black/white. IF you can apply a statute - dont need to look to RPP. Violation of statute = shortcut if statute is applicable. The D owed me a duty to drive like a RPP v. The D owned me a duty to stop at a red light bc statute says so. Problem is that the statute says pay $200 (not that ur negligent). So using statute in a way that it wasnt meant to be used. In essence Borrowing a statute from crim law and asking judge to use it as tort neg standard A. "Negligence per se " doctrine: Most courts apply the "negligence per se" doctrine: when a safety statute has a sufficiently close application to the facts of the case at hand, an unexcused violation of that statute by D is "negligence per se," and thus conclusively establishes that D was negligent. Its negligence per se bc there is no evaluatory function (circumstances dont matter no RPP under the circumstances). NEG per se cancels out duty/breach. STILL HAVE TO PROVE CAUSATION/damages. Jury will still consider the FACTS - was the light red or green. B. P has to show he is a member of the class of the a. a member of the class of the Same Class of persons the crim statute was designed to protect b. Same Type of harm statute is designed to protect Example: D drives at 65 m.p.h. in a 55 m.p.h. zone. While so driving, he strikes and injures P, a pedestrian. Because the 55 m.p.h. limit is a safety measure designed to protect against accidents, the fact that D has violated the statute without excuse conclusively establishes that D was negligent D will not be permitted to argue that it was in fact safe to drive at 65 m.p.h. Osborne v. McMasters - If a person neglects to perform a duty imposed by either statute or common law and that law is designed for the protection of others, then the evidence of the act or omission is negligence per se

B. Statute must apply to facts: The negligence per se doctrine will apply only where P shows that the statute was intended to guard against the very kind of injury in question. 1. Class of persons protected: This means that P must be a member of the class of persons whom the statute was designed to protect. (Example: A statute requires all factory elevators to be provided with a certain safety device. The legislative history shows that the purpose was only to protect injuries to employees. P, a business visitor, is injured when the elevator falls due to lack of the device. P cannot use the negligence per se doctrine, because he was not a member of the class of persons whom the statute was designed to protect.) Stachneiwicz v. Mar-Cam Corp P was injured during a barroom brawl The regulation was drafted to prevent injuries and abuses associated with the types of disturbances connected with bars and intoxication. Because P was within the class of persons the regulation sought to protect and because the injuries he sustained were of the type of harm the regulation sought to avoid, the violation of the regulation can be used as evidence of negligence per se. 2. Protection against particular harm: Second, the statute must have been intended to protect against the particular kind of harm that P seeks to recover for. (Example: A statute requires that when animals are transported, each breed must be kept in a separate pen. D, a ship operator, violates the statute by herding Ps sheep together with other animals. Because there are no pens, the sheep are washed overboard during a storm. P cannot use the negligence per se doctrine, because the statute was obviously intended to protect only against spread of disease, not washing overboard. [Gorris v. Scott, ]) Example 2: Ney v. Yellow Cab Co. Link between purpose of statute an prox cause = FORSEEABILITY If the harm resulting from negligence is foreseeable, then an intervening criminal act does not break the chain of causation. What is purpose of statute safety or anti-theft? Example 3: Perry v. S.N. and S.N. - Statute is spelling out standard in particular factual conduct. The court decides that the statute does not establish a standard of conduct. Thus, it finds that the plaintiffs failed to state a cause of action. Some statutes merely codify common law duties. However, there is no common law duty to report child abuse EXCEPTIONS: (1) D was reasonably unaware of the particular occasion for compliance; - (Example: A statute requires all brakes to be maintained in good working order. Ds brakes fail, and he cant stop, so he runs over P. If D can show that he had no way

to know that his brakes were not in working order, his violation of the statute would be excused, and the negligence per se doctrine will not apply. [Freund v. DeBuse, ]) (2) D made a reasonable and diligent attempt to comply; (3) D was confronted with an emergency not of his own making; - heart attack while driving and runs a red light. Again must revert to RPP standard. (might not be any negligence if sudden onset but a RPP might stop driving when got chest pains). (4) compliance would have involved a greater risk of harm. (Example: swerve to miss a kid and cross double yellow into other lane and hit a car. cant use statute as subs. for standard of care and instead must use RPP standard). (Example: P violated statute requiring Pedestrians to walk on the sidewalk but if she did she would most likely fall Zeni v. Anderson) (5) Compliance was impossible under the circumstances 4. Contributory negligence per se : If the jurisdiction recognizes contributory negligence, D may get the benefit of contributory negligence per se where P violates a statute. (Example: Cars driven by P and D collide. If P was violating the speed limit, and the jurisdiction recognizes contributory negligence, D can probably use the negligence per se doctrine to establish that P was contributorily negligent.) 5. Compliance not dispositive: The fact that D has fully complied with all applicable safety statutes does not by itself establish that he was not negligent the finder of fact is always free to conclude that a reasonable person would take precautions beyond those required by statute. Its just mere evidence Merely evidence: in some J violation of a stat is merely evidence of neg and in others its neg per se. When kids violate statutes it is MERELY EVIDENCE of negligence not neg per se

DUTY GENERALLY- special rules that have to do w duty.


Concept: Generally, a person owes everyone else with whom he comes in contact a general "duty of care." Normally, you dont have to worry about this duty it is the same in all instances, the duty to behave with the care that would be shown by a reasonable person. You dont have to drive a car/open a restaurant/go skiing but if you do do it carefully. But there are several situations in which courts hold that the defendant owes plaintiff less than this regular duty. The most important of these situations are: (1) D generally has no duty to take affirmative action to help P; (2) D generally has no duty to avoid causing unintended mental suffering to P; and (3) D has no duty to avoid causing pure economic loss to P in the absence of more tangible types of harm such as physical injury.

FAILURE TO ACT
A. No general duty to act: A person generally cannot be liable in tort solely on the grounds that she has failed to act. 1. Duty to protect or give aid: This means that if D sees that P is in danger, and fails to render assistance (even though D could do so easily and safely), D is not liable for refusing to assist. (Example: D even Michael Phelps, passing by, sees P drowning in a pond. D could easily pull P to safety without risk to D, but instead, D walks on by. D is not liable to P.) B. Exceptions: PRE-EXISTING Special Relationships: position of superiority/inferiority where 1 person has control. Look for formal relationship recognized by the legal system Common Carrier/passenger (bus driver has to rescue passenger) Innkeeper/Guest o Business Owner/patron: Example: L.S. Ayres Little boy gets caught in escalator. Blame Department store for taking too long in turning escalator off - which P's injuries were aggravated - D has DUTY not to further aggravate injuries) o Employers and universities: Similarly, employers must give assistance to employees, and universities must give assistance to students. (BUT - Hegel v. Langsam -Uni has NO DUTY to regulate the private lives of their students.) Legal Custodian. Prison Guard/prisoner. Custodian has control - prisoner has no ability to help themselves. Land owner/occupier/invitee

Bar owners - must allow good Samaritan to use public phone on premises (its so minimal a burden - you have to allow) Husband/wife and parents/kids Doctor/Patient relationship: Dr has a DUTY to exercise reasonable care to protect the foreseeable victim of that danger - Tarasoff v. uni of CA Psychologist councils guy who tells him he will kill - detains him - releases him - and then he kills the woman o Jail/jailor 2. Defendant CAUSED the Ps peril: If the danger or injury to P is due to Ds own conduct, or to an instrument under Ds control, D has the duty of assistance. This is true today even if D acted without fault. (Example: A car driven by D strikes P, a pedestrian. Even though D has driven completely non-negligently, and the accident is due to Ps carelessness in crossing the street, D today has a common-law duty to stop and give reasonable assistance to P.) 3. Defendant and victim as co-venturers: Where the victim and the defendant are engaged in a common pursuit, so that they may be said to be co-venturers, some courts have imposed on the defendant a duty of warning and assistance. For instance, if two friends went on a jog together, or on a camping trip, their joint pursuit might be enough to give rise to a duty on each to aid the other. ***** Once a duty is established The duty in all these is RPP under the circumstances***** - This never Requires D to put their own life in jeopardy. Throw a rope dial 911 4. Assumption of duty: Once D voluntarily begins to render assistance to P (even if D was under no legal obligation to do so), D must proceed with reasonable care. A gratuitous rescuer will be liable for botching the rescue. If you dont want to rescue you dont have to but if you choose to rescue you have to do it as a RPP. You do the Heimlich but are a little tipsy and break the Ps ribs liable. *** Good Samaritan Law: some states reverse CL rule a gratuitous rescuer gets some immunity from act a. Preventing assistance by others: D is especially likely to be found liable if he begins to render assistance, and this has the effect of dissuading others from helping P. (Example: If D stops by the roadside to help P, an injured pedestrian, and other passers-by decline to help because they think the problem is taken care of, D may not then abandon the attempt to help P.) b. Mere promise: Traditionally, a mere promise by D to help P (without actual commencement of assistance) was not enough to make D liable for not following through. But many modern courts would make D liable even in this situation, if P has a reliance interest. 5. Duty to control others: If D has a duty to control third persons, D can be negligent for failing to exercise that control. a. Special relationship: A duty to control a third person may arise either because of a special relationship between D and P, or a special relationship between D and a third person. For instance, some courts now hold that any business open to the public must protect its patrons from wrongdoing by third parties. (Example: D, a storekeeper, fails to take action when X, an obviously deranged man, comes into the store wielding a knife. P, a patron, is stabbed. Most courts would find D liable for failing to take action.) Example2: JS and MS v RTH a wife who suspects or should suspect her husband of actual or prospective sexual abuse of their neighbors children has any duty of care to prevents such abuse Wife is in a unique position to look for the signs. Empirical evidence to fit In w foreseeability analysis Particularized Foreseeability: wife might have special reason to know of abuse Proximate Cause: failure to report causes continuation of abuse. If she does not report - she breached that duty. The harm is DIRECT and PREDICTABLE

PURE ECONOMIC LOSS limited duty


A. Traditional rule: Where D tortuously causes physical injury or property damage to X, but only pure economic loss to P, the traditional rule is that P may not recover anything. (Example: A ship owned by D damages a dock owned by X. P, owner of a different ship, is required to dock elsewhere and suffers extra labor and docking costs, but no physical injury. Under the traditional view, P may not recover these expenses from D, even though D was a tortfeasor vis-a-vis X. Rationale: The rationale for this restrictive rule is the fear of open-ended liability. B. Modern approach: But most modern courts probably no longer impose a blanket rule of no liability for pure economic loss. If a court does decide to relax the no-liability rule, it is most likely to award recovery where: (1) the injury

to P was relatively foreseeable; (2) relatively few plaintiffs would be permitted to sue if liability were found for pure economic loss; and (3) Ds conduct is relatively blameworthy. Example: State of Louisiana ex rel. Guste v. M/V Testbank - Two ships collided, causing toxic chemicals to be released into the environment and causing the United States Coast Guard (Coast Guard) to close a Mississippi River outlet Rule of Law. The majority rule does not allow recovery for economic loss, unless physical damage to property occurs EXCEPTION for COMMERCIAL FISHERMAN: oysterman can recovery. Their property is the fish BP Spill: Could recover if you boat was damaged, but not for economic loss - if you lost business/jet ski Court relies on rule from ROBBINS Case: limit liability to those who you are in a contractual relationship w. POLICY: worried about extended liability /millions of dollars/ Courts effort to limit liability. Duty LIMITED DUTY:

IV. Emotional Distress


A. Accompanied by physical impact: If D causes an actual physical impact to Ps person, D is liable not only for the physical consequences of that act but also for all of the emotional or mental suffering which flows naturally from it. Such mental-suffering damages are called "parasitic" they attach to the physical injury. ($$ bc cant walk daughter down the isle etc. ) This is not what were talking about right now.... STAND ALONE DISTRESS: B. Mental suffering without physical impact: But where there has been no physical impact or direct physical injury to P, courts limit Ps right to recover for mental suffering. Daley v. LaCroix the common law impact requirement should no longer apply in Michigan D, wrecked his car shearing off a utility pole and creating an electrical explosion at Daleys house - claimed, in addition to property damage, that two family members suffered severe emotional disturbance as a result of the accident * Several limitations are applicable to this new holding. First, defendants standard of conduct is measured by reactions to be expected by RPP Second, plaintiff has the burden of proof that the physical harm complained of is a natural consequence of the alleged emotional disturbance, proximately caused by defendants conduct. 3 cases you can recover: 1. No physical symptoms: Where there is not only no impact, but no physical symptoms of the emotional distress at all, nearly all courts deny recovery. (Example: D narrowly misses running over P. No one is hurt. P has no physical symptoms, but is distraught for weeks. Few if any courts will allow P to recover for her emotional distress.) a. The "at risk" plaintiff: The general rule means that if P, by virtue of his exposure to a certain substance, suffers an increased likelihood of a particular disease, P may generally not recover for the purely emotional harm of being at risk. (Example: D releases toxic chemicals into the water. This causes P to have a greatly increased risk of throat cancer. Most courts will not allow P to recover for distress at being extra vulnerable to cancer.) b. Intentional torts: Remember that the general rule applies only to negligent conduct by D if Ds conduct is intentional or willful, P may recover for purely emotional harm with no physical symptoms, by use of the tort of intentional infliction of emotional distress. EXCEPTIONS: Some courts recognize an exception to this rule in special circumstances (e.g., negligence by telegraph companies in wording messages, and by funeral homes in handling corpses). Special Relationship (Dr./Patient) and highly foreseeable result of carelessness (Dr. mixes up results and tell u you have HIV ) can collect. 2. NEAR MISS: Physical injury without impact: Where Ds negligent act (1) physically endangers P, (2) does not result in physical impact on P, and (3) causes P to suffer emotional distress that has physical consequences (Heart Attack/miscarriage/rash/tremor) nearly all courts allow recovery. (Example: D narrowly avoids running over P. P is so frightened that she suffers a miscarriage. P may recover.) More and more liberal trend. Clinical depression may reach the standard

3. BYSTANDER: P witnesses a severe negligent injury to a close family member and P suffers purely emotional distress (without physical consequences), and Ps distress is due solely to grief. Eyewitness to trajedy alters your life forever. a. Zone of danger: If P was in the "zone of danger" (i.e., physically endangered but not struck), nearly all courts allow him to recover for emotional distress due to another persons plight. (Exampl e: D narrowly avoids running over P, and in fact runs over Ps child S. Most courts will allow P to recover for her emotional distress at seeing S injured.) b. Abandonment of zone requirement: A number of states probably still a minority have abandoned the "zone of danger" requirement. Thing v. La Chusa - Contain liability via bright line rule P may recover damages for emotional distress caused by observing the negligently inflicted injury of a third person only if: (1) the plaintiff is closely related to the victim; (broad/narrow interp. By J) (2) the plaintiff is present at the scene at the time the injury occurs and is then aware of the injury being caused to the victim; (some states allow recovery if you come on scene shortly after it happened). (3) as a result suffers serious emotional distress beyond which would be expected of a disinterested witness. In the present case, Plaintiff was not present at the scene of the accident, therefore, she cannot establish a right to recover for her emotional distress. LIMITED DUTY:

IV. UNBORN CHILDREN- duty towards the unborn


A. Modern view: Most courts have rejected the traditional view that an infant injured in a pre-natal accident could never recover if born alive. Today, recovery for pre-natal injuries varies: 1. Child born alive: If the child is eventually born alive, nearly all courts allow recovery. (Example: D makes a drug taken by Ps mother while P is a fetus only a few weeks old. P is born with serious birth defects resulting from the drug. Nearly all courts would allow P to recover.) 2. Child not born alive: Courts are split about whether suit can be brought on behalf of a child who was not born alive. Usually, a court will allow recovery only if it finds that a fetus never born alive is a "person" for purposes of the wrongful death statute. Mrs. Endresz was seven months pregnant when she was injured in an automobile accident with Friedberg, causing her twins to be delivered stillborn 3. Pre-conception injuries: The above discussion assumes that the injury occurred while the child was in utero. Suppose, however, that the injury occurred before the child was even conceived, but that some effect from the injury is nonetheless suffered by the later-conceived child. Here, courts are split as to whether the child may recover. (Example: Ps mother, before getting pregnant with P, takes a drug made by D. The drug damages the mothers reproductive system. When P is conceived, she suffers from some congenital disease or defect (e.g., sterility) as a result. Ps mother can clearly recover from D for her own injuries, but courts are split as to whether P can recover against D for these pre-conception events. [Enright v. Eli Lilly]) 4. Wrongful life: If a child is born illegitimate, or with an unpreventable congenital disease, the child may argue that it should be entitled to recover for "wrongful life," in the sense that it would have been better off aborted. But almost no courts have allowed the child to make such a wrongful life recovery. Courts do, however, often allow the parents to recover for their medical expenses, and perhaps their emotional distress from the childs condition EXAMPLE: Procanik by Procanik v. Cillo - The Plaintiff was born with multiple birth defects, the result of his mothers German Measles that the dr. negligently failed to diagnose. Rule: An infant plaintiff may recover special damages for the extraordinary medical expenses he will incur, but may not recover general damages for emotional distress or for an impaired childhood. * Policy considerations have led this court to decline to recognize any cause of action for an infants wrongful life. (Really hard to quantify) The problem is that the Plaintiff claims not that he should not have been born with defects, but rather that he should not have been born at all. This Court finds that there is no rational way to compare a lack of existence with the pain and suffering of Plaintiffs impaired existence in order to all recovery for his emotional distress or diminished childhood. However, this Court does find that recovery of the cost of extraordinary medical expenses is

recoverable by either the parents or the infant, but not both. The infant can wait until his majority to recover medical expenses.

Unforeseeable plaintiff:
No duty to the unforeseeable P: dont owe a duty to someone who is very far away/an unforeseeable victim The general rule that D is liable only for foreseeable consequences is also usually applied to the "unforeseeable plaintiff" problem. That is, if Ds conduct is negligent as to X (in the sense that it imposes an unreasonable risk of harm upon X), but not negligent as to P (i.e., does not impose an unreasonable risk of harm upon P), P will not be able to recover if through some fluke he is injured. Example: X, trying to board Ds train, is pushed by Ds employee. X drops a package, which (unknown to anybody) contains fireworks, which explode when they fall. The shock of the explosion makes some scales at the other end of the platform fall down, hitting P. Held, P may not recover against D. Ds employee may have been negligent towards X (by pushing him), but the employees conduct did not involve any foreseeable risk of harm to P, who was standing far away. Since Ds conduct did not involve an unreasonable risk of harm to P, and the damage to her was not foreseeable, the fact that the conduct was unjustifiably risky to X is irrelevant. Ds conduct was not the "proximate cause" of the harm to P. [Palsgraf v. Long Island R.R. Co., ] Since the conductors would not have anticipated injury to P from their conduct, they owed not duty to avoid the injury and were not negligent in relation to her. ** Another way to look at this is that there is no transferred negligence CARDOZO v. ANDREWS Cardozo: Majority: Cardozo's opinion says that a duty should be extended only to those who could be reasonably foreseen to be within the scope of danger if a person were to act negligently. You cannot have a duty to those whom you cannot perceive to be in danger of your actions, or as he says, "The risk reasonably to be perceived defines the duty to be obeyed and risk imports relation; it is risk to another or to others within the range of apprehension." unforeseeable p = YOU DONT OWE A DUTY TO SOMEONE WHOSE FAR AWAY/unforeseeable victim P was unable to demonstrate a right to recovery bc she was an unforeseeable victim. Foreseeability is what would I have expected to happen at the moment of the breach. Who are the ppl who might get hurt? It is not foreseeable that someone way down duty analysis = judge we can stop at duty and dont need to go to breach/cause/damages. BC there is no duty you are done w analysis Dissent: Andrews says that people have duties to society as a whole, and if one is negligent, then a duty existed no matter what. Andrews introduced the concept of the zone of danger. If youre within the zone of danger created by the defendants negligent conduct, then the defendant will be liable for your harm. However, Andrews does believe that negligence can be cut off via proximate cause, and an actor is only liable for the damages that resulted out of his negligence. Andrews prox cause = jury you have to demonstrate you were a foreseeable victim of the breach in Q. If an act unreasonable threatens the safety of others, the doer is liable for all proximate consequences, even where the injury is thought by the RPP to be outside radius of danger. He defines the relationship more broadly. If you hurt someone/harm them that relationship is formed. You harmed them - you're liable. Whether they were standing right there or a mile away - you harmed them. The ANDREWS theory 1. Direct or remote harm. Time and space 2. Natural or continuous sequence of events 3. Substantial factor in causing harm And applying these factors, there is a direct relationship w Palsgraf (or it should at least go to the jury) So why the splitting of hairs? It all comes down to the connection between proximate cause and duty. Both serve as a means of cutting off a defendant's liability, but they operate in slightly different manners. And the difference is not a mere formality, because the judge determines duties and the jury determines proximate cause. Andrews is not saying

that D is liable to P; he is simply arguing that a duty existed and that duty was breached, and thus it is up to the jury to decide if his actions were the proximate cause of the injury.

NEGLIGENCE BREACH:
Identification of wrong behavior. P will claim that D breached this duty by QUOTE FROM FACT PATTERN. This is unreasonable Reasonable people keep their eyes on the road. ID the conduct and then explain why that conduct falls short of the standard. Conduct + reason.

because

APPLY STANDARD OF DUTY UNDER BREACH:


US v. Carroll Towing Co. HAND FORMULA - Learned Hand says that the owners duty depends on three variables: a. FORSEEABLE HARM: How likely is it that the barge will break away from its moorings? b. FORESEEABLE SEVERITY OF ANY HARM: How big an injury did it cause? THE BURDEN OF PRECAUTIONS TO ELIMINATE/REDUCE RISK OF HARM: How expensive would precautions be? - Access the Burden: 2. Social value 3. Relative safety of alternative conduct If the expense of precautions is less the expectation value of the injury, then the court will conclude that the owner had a duty that was breached. B PL Liability depends upon whether B (Burden) is less that L (the injury) times P (Probability) Liability when: B<PL Example: (applies to rough estimates, hard numbers are not necessary) D could purchase avalanche beacon for $100. Not having the beacon increases the chance of accident by 25%. The magnitude of accident, if it occurs is $1,000. B = $1000; P = .25; L = $1,000 B($100) <PL($250), So Beacon IS required. Pros: Places risks & utilities on common scale for comparison. Gives incentives to invest an appropriate amount in safety. Protects risks that are worth taking (for society) Requires that courts be able to explain their conclusions about negligence. Cons: Critique that it does not provide incentives. Sacrifices individual good: too much emphasis on social good & not enough on individual justice. Dollar scale simply doesnt work for loss of life measuring incommensurables.

Proof of Breach - CIRCUMSTANTIAL EVIDENCE:


Banana peel cases
Circumstantial Evidence: requires an inference or additional reasoning to reach desired matter at hand

P needs to prove 1. D had Actual/constructive notice prove with time and other circumstances a. the banana peel is brown/flat = its been there a while. They knew or should have known b. the cleanup schedule = evidence c. jury gets to decide 2. The condition posed an unreasonable risk of harm
Duty is to exercise reasonable care to keep premises safe Manner/Method of Sale: merchant already on notice bc of the manner in which something is sold

EXAMPLE: P slips on Pizza on terrazzo floor - P argues she doesnt need to show constructive

notice bc their method of serving means its always dangerous (serving pizza on wax paper to customers who are standing) D recognize this bc they're always cleaning the floor - NOTICE NOT REQUIRED IF DANGER IS CONTINUOUS 3. D did not exercise reasonable care 4. D's failure to care caused the injury Burden is on P bc otherwise all P would need to prove is that the accident happened

Proof of Breach - RES IPSA LOQUITUR


No evidence =res ipsa - Not required to show 100% certainty. Preponderance of the evidence 51% A. Generally: The doctrine of res ipsa loquitur ("the thing speaks for itself") allows P to point to the fact of the accident, and to create an inference that, even without a precise showing of how D behaved, D was probably negligent. Example: A barrel of flour falls on Ps head as he walks below a window on the street. At trial, P shows that the barrel fell out of a window of Ds shop, and that barrels do not fall out of windows without some negligence. By use of the res ipsa loquitur doctrine, P has presented enough evidence to justify a verdict for him, so unless D comes up with rebuttal evidence that the barrel did not come from his shop or was not dropped by negligence, D will lose. [Byrne v. Boadle] B. Requirements for: Courts generally impose four requirements for the res ipsa doctrine: 1. No direct evidence of Ds conduct: There must be no direct evidence of how D behaved in connection with the event. Be an accident 2. Seldom occurring without negligence: (Wouldnt have happened if someone was applying due care/RPP) P must demonstrate that the harm which occurred does not normally occur except through the negligence of someone. P only has to prove that most of the time, negligence is the cause of such occurrences. Example: Normally a plane does not fall from the sky there was an error. Its an appeal to common understanding. P will generally be able to establish that airplanes usually do not crash without some negligence, thus meeting this requirement. 3. Exclusive control of defendant: P must demonstrate that the instrumentality which caused the harm was at all times within the exclusive control of D. (Example: P, while walking on the sidewalk next to D hotel, is hit by a falling armchair. Without more proof, P has not satisfied the "exclusive control" requirement, because a guest, rather than the hotel, may have had control of the chair at the moment it was dropped. [Larson v. St. Francis Hotel]) a. Multiple defendants: If there are two or more defendants, and P can show that at least one of the defendants was in control, some cases allow P to recover. This is especially likely where all of the Ds participate together in an integrated relationship. (Example: P is injured while on the operating table, and shows that either the surgeon, the attending physician, the hospital, or the anesthesiologist must have been at fault, but is unable to show which one. P gets the benefit of res ipsa, and it is up to each individual defendant to exculpate himself. [Ybarra v. Spangard] = There is NO OTHER WAY this happened except via negligence) 4. Not due to plaintiff: P must establish that the accident was probably not due to his own conduct. 5. Evidence more available to D: Some courts also require that evidence of what really happened be more available to D than to P. (Example: This requirement is satisfied on the facts of Ybarra, supra, since the Ds obviously knew more than the unconscious patient about who was at fault.) C. Effect of res ipsa : Usually, the effect of res ipsa is to permit an inference that D was negligent, even though there is no direct evidence of direct breach negligence. Res ipsa thus allows a particular kind of circumstantial evidence. When res ipsa is used, P has met his burden of production, and is thus entitled to go to the jury. Gets you past summary judgment. D. Rebuttal evidence: 1. General evidence of due care: If Ds rebuttal is merely in the form of evidence showing that he was in fact careful, this will almost never be enough to give D a directed verdict the case will still go to the jury. 2. Rebuttal of res ipsa requirements: But if Ds evidence directly disproves one of the requirements for the doctrines application, then D will get a directed verdict (assuming there is no prima facie case apart from res ipsa). (Example: If D can show that the instrument that caused the harm was not within his control at all relevant times, the doctrine will not apply, and D may get a directed verdict.)

Example: Sullivan v. Crabtree - The doctrine of Res Ipsa Loquitur creates a burden on the defendant to show a reasonable explanation for the injury. The strength of that burden depends on the facts of each case and the strength of the inference created.

CAUSATION
NEGLIGENCE ACTUAL AND PROXIMATE CAUSE I. Causation in Fact
A. Generally: P must show that Ds conduct was the "cause in fact" of Ps injury. A linkage. B. "But for" test: The vast majority of the time, the way P shows "cause in fact" is to show that Ds conduct was a "but for" cause of Ps injuries had D not acted negligently, Ps injuries would not have resulted. But for the breach, P would be uninjured today. Ds are not causes breaches are causes. Example: A statute requires all vessels to have life boats. D sends out a boat without life boats. P, a sailor, falls overboard in a storm so heavy that, even had there been a life boat, it could not have been launched. P drowns. Even assuming that D was negligent per se, Ds failure to provide life boats is not a cause in fact of Ps death, because that death would have occurred even without the failure. Therefore, D is not liable. Example2: Perkins v. Railroad - Even if train was following speed limit - the accident would have happened anyways no causation - They killed him - but they weren't negligent - Can use speed limit to prove negligence per se, but not that it caused the harm Example3: Reynold v. Railroad - Negligence of Df greatly multiplies the chances of accident to the Pl and is of a character naturally leading to it occurrence, the mere possibility that it might have happened without the negligence is not sufficient to break the chain of cause and effect between the negligence and the injury. The evidence connects the accident with the negligence. Example4: Gentry v. Douglas Hereford Ranch - Bad husband who hunts instead of paint- No clear way to establish WHAT caused his gun to go off cant prove causation - Causation requires proof of proximate cause and cause in fact for liability to attach Example5: Kramer Service v. Wilkins - P cut his head on some glass in a hotel, and it was clear that the hotels negligence was the cause of that harm. the exact cause of the cancer remains UNKNOWN (1939) The plaintiff must prove as an element of the negligence cause of action that the defendants negligence was the actual cause of the harm done to the plaintiff. That its POSSIBLE - is not enough for causation

MULTIPLE DEFENDENTS: The but for test doesnt work when two actors are negligent and act at the same time. 1. Joint tortfeasors: There can be multiple "but for" causes of an event. D1 cannot defend on the grounds that D2 was a "but for" cause of Ps injuries as long as D1 was also a "but for" cause, D1 is viewed as the "cause in fact." C. Merged /Concurrent causes: Sometimes Ds conduct can meet the "cause in fact" requirement even though it is not a "but for" cause. This happens where two events concur to cause harm, and either one would have been sufficient to cause substantially the same harm without the other. Each of these concurring events is deemed a cause in fact of the injury, since it would have been sufficient to bring the injury about. (Example: Sparks from Ds locomotive start a forest fire; the fire merges with some other unknown fire, and the combined fires burn Ps property. Either fire alone would have been sufficient to burn Ps property. Therefore, Ds fire is a cause in fact of Ps damage, even though it is not a "but for" cause. [Kingston v. Chicago & N.W. Ry.]) ( Hill v. Edmonds - RULE: Where separate acts of negligence combine to produce directly a single injury each tortfeasor is responsible for the entire result, even though his act alone might not have caused it.) Substantial factor test Anderson v. Minneapolis St. P. & S. St. M. R.R. Co. - Say there is a negligent actor and an innocent actor. A fire that was started negligently by the defendant combined with a fire of unknown origin. Is the railroad liable? Is the defendants fire a but for cause of the plaintiffs injury? Clearly no. We dont know which fire did it, so how do we decide who to hold responsible? In this case, the defendant is found 100% responsible. Is each breach by itself capable of causing the injury. In 2 fires yes. If 2 polluters would depend on how much haz waste each polluter dumped into river.

To be liable - whichever was first is liable. If fire b destroyed house and fire A came along and hour later, fire A can't be liable D. Multiple fault: There forces did not combine. If P can show that each of two (or more) defendants was at fault, but only one could have caused the injury, the burden shifts to each defendant to show that the other caused the harm they should fight it out amongst themselves. If no one can prove the other did it - then they're all liable (Example: P, D1 and D2 go hunting together. D1 and D2 simultaneously fire negligently, and P is struck by one of the shots. It is not known who fired the fatal shot. The court will put the burden on each of the Ds to show that it was the other shot which hit P if neither D can make this showing, both will be liable. [Summers v. Tice]) 1. The "market share" theory: In product liability cases, courts often apply the "market share" theory. If P cannot prove which of three or more persons caused his injury, but can show that all produced a defective product, the court will require each of the Ds to pay that percentage of Ps injuries which that Ds sales bore to the total market sales of that type of product at the time of injury. The theory is used most often in cases involving prescription drugs. Example: 200 manufacturers make the drug DES. P shows that her mother took the drug during pregnancy, and that the drug caused P to develop cancer. P cannot show which DES manufacturer produced the drug taken by her mother. Held, any manufacturer who cannot show that it could not have produced the particular doses taken by Ps mother will be liable for the proportion of any judgment represented by that manufacturers share of the overall DES market. *Sindell v. Abbott Laboratories,] a. Exculpation: Courts are split on whether each defendant should be allowed to exculpate itself by showing that it did not make the particular items in question some more modern cases hold that once a given defendant is shown to have produced drugs for the national market, no exculpation will be allowed. b. National market share: In determining market share, courts usually use a national, rather than local, market concept c. No joint and several liability: Courts adopting the "market share" approach often reject jointand-several liability they allow P to collect from any defendant only that defendants proportionate share of the harm caused. (Example: P sues a single D, and shows that that D counted for 10% of the market. Ps total damages are $1 million. If "market share" is the theory of liability, most courts will allow P only to recover $100,000 from D D will not be made jointly and severally liable for Ps entire injuries.) d. Socially valuable products: The more socially valuable the court perceives the product to be, the less likely it is to apply a market-share doctrine. For instance, a court is likely to reject the doctrine where the product is a vaccine.

Herskovitz v. Group Health Cooperative of Puget Sound Daubert v. Merrell Dow Pharmaceuticals, Inc.
COME BACK TO THESE

II. PROXIMATE CAUSE (scope of the risk)


Types of Cases - Foreseeability comes up in 4 ways 1. Unforeseeable P Polsgraf (under duty) 2. Unforeseeable Type of Harm a. Wagon mound limit liability to foreseeable type of harm b. 3rd Restatement 29 must be within the scope of risk 3. Unforeseeable extent of harm - Bartolone - eggshell skull P too bad for D 4. Unforeseeable Manor of Harm - Type of harm may have been Foreseeable - but not the precise manor which it came out - too bad Public policy doctrine courts cut off liability

1. direct causation Polemis/wagon mound 1. Wagon overruled Polemis now only liable for foreseeable results 2. indirect some intervening force A. General: Even after P has shown that D was the "cause in fact" of Ps injuries, P must still show that D was the "proximate cause" of those injuries. P MUST SHOW THAT HE WAS A FORESEEABLE VICTIM WHO SUSTAINED A FORESEABLE TYPE OF HARM. Everyone is liable for the proximate (immediate) results of their own acts. The proximate cause requirement is a policy determination that a defendant, even one who has behaved negligently, should not automatically be liable for all the consequences, no matter how improbable or far-reaching, of his act. Today, the proximate cause requirement usually means that D will not be liable for the consequences that are very unforeseeable. Example: D, driving carelessly, collides with a car driven by X. Unbeknownst to D, the car contains dynamite, which explodes. Ten blocks away, a nurse who is carrying P, an infant, is startled by the explosion, and drops P. P will not be able to recover against D, because the episode is so far-fetched it was so unforeseeable that the injury would occur from Ds negligence that courts will hold that Ds careless driving was not the "proximate cause" of Ps injuries. Example: Ds ship spills oil into a bay. Some of the oil adheres to Ps wharf. The oil is then set afire by some molten metal dropped by Ps worker, which ignites a cotton rag floating on the water. Ps whole dock then burns. Held, D is not liable, because the burning of Ps dock was not the foreseeable consequence of Ds oil spill, and thus the oil spill was not the proximate cause of the damage. This is true even though the burning may have been the "direct" result of Ds negligence. *Wagon Mound No. 1] 1. Multiple proximate causes: Just as an occurrence can have many "causes in fact," so it may well have more than one proximate cause. (Example: Each of two drivers drives negligently, and P is injured. Each driver is probably a proximate cause of the accident.)

2. General type of harm is foreseeable - IT DOES NOT MATTER the way the harm came about (just like in prac exam) 3. See 3rd R shift to scope of risk analysis gaining ground. Limiting liability to type of harm w in scope of risk. 29 1. Hunter hands girl gun she brakes toe not liable
OLD WAY: The Polemis test is: 1. Was the defendant negligent? 2. Is the defendants negligence a direct cause of the damages? ***Under this case, you have liability no matter how remote the consequences of ones negligence are. This is no longer the current test, but it is important to know. It is an alternative to the foreseeability analysis of Wagon Mound and Palsgraf. Under Polemis, Wagon Mound No. 1 would have come out differently. Instead, the court adopted a new test: Ex ante, before the accident happens, what would a reasonable person foresee as the kinds of harms that might occur stemming from that negligent conduct? An explosion due to a negligently carried plank is not foreseeable harm. Thus, by the rule of Wagon Mound No. 1, Polemis would have gone the other way. In Wagon Mound No. 1, you can look at the circumstances surrounding the accident to find out if the risk was really foreseeable. Wagon Mound No. 2 comes out a different way based on different lawyering. What would be the result of this case under Polemis? There was a natural sequence of events leading to Mrs. Palsgrafs injury. Therefore, under the old rule, the railroad would be liable.

EXCEPTION TO UNFORESEEABLE: B. Extensive consequences from physical injuries: TAKE P AS YOU FIND HIM:A key exception to the general rule that D is liable only for foreseeable consequences is: once P suffers any foreseeable impact or injury, even if relatively minor, D is liable for any additional unforeseen physical consequences.

1. Egg-shell skull: Thus if P, unbeknownst to D, has a very thin skull (a skull of "egg-shell thinness"), and D negligently inflicts a minor impact on this skull, D will be liable if, because of the hidden skull defect, P dies. The defendant "takes his plaintiff as he finds him." Example: Bartolone v. Jeckovich - P was in a car accident w minor injuries but afterwards had sort of an acute psychotic breakdown. - The court finds that the plaintiff had a pre-existing condition such that he was normal until the accident, but the accident pushed him over the edge into debilitating mental illness. The court says that it is thus appropriate for the jury to find the defendant responsible for this disability. D. General class of harm but not same manner: Another exception to the "foreseeable consequences only" rule is that as long as the harm suffered by P is of the same general sort that made Ds conduct negligent, it is irrelevant that the harm occurred in an unusual manner. Example: D gives a loaded pistol to X, an eight-year-old, to carry to P. In handing the pistol to P, X drops it, injuring the bare foot of Y, his playmate. The fall sets off the gun, wounding P. D is liable to P, since the same general kind of risk that made Ds conduct negligent (the risk of accidental discharge) has materialized to injure P; the fact that the discharge occurred in an unforeseeable manner by the dropping of the gun is irrelevant. (But D is not liable to Y, since Ys foot injury was not foreseeable, and the risk of it was not one of the risks that made Ds conduct initially negligent.) Direct Harm: when the harm is instantaneous/directly after the act it almost always is foreseeable. Run read light and hit ped and break his leg. Where it gets interesting is the gap in time. Then the injury may have been unforeseeable.

IV. PROXIMATE CAUSE INTERVENING CAUSES


A. Definition of "intervening cause": Most proximate cause issues arise where Ps injury is precipitated by an "intervening cause." An intervening cause is a force which takes effect after Ds negligence, and which contributes to that negligence in producing Ps injury. 1. Superseding cause: Some, but not all, intervening causes are sufficient to prevent Ds negligence from being held to be the proximate cause of the injury. Intervening causes that are sufficient to prevent D from being negligent are called "superseding" causes, since they supersede or cancel Ds liability. B. Foreseeability rule: Generally courts use a foreseeability rule to determine whether a particular intervening cause is superseding. 1. Test: If D should have foreseen the possibility that the intervening cause (or one like it) might occur, or if the kind of harm suffered by P was foreseeable (even if the intervening cause was not itself foreseeable), Ds conduct will nonetheless be the proximate cause. But if neither the intervening cause nor the kind of harm was foreseeable, the intervening cause will be a superseding one, relieving D of liability. C. Foreseeable intervening causes: Often the risk of a particular kind of intervening cause is the very risk (or one of the risks) which made Ds conduct negligent in the first place. Where this is the case, the intervening cause will almost never relieve D of liability. Example: D leaves his car keys in the ignition, and the car unlocked, while going into a store to do an errand. X comes along, steals the car, and while driving fast to get out of the neighborhood, runs over P. If the court believes that the risk of theft is one of the things that makes leaving ones keys in the ignition negligent, the court will almost certainly conclude that Xs intervening act was not superseding. 1. Foreseeable negligence: The negligence of third persons may similarly be an intervening force that is sufficiently foreseeable that it will not relieve D of liability. (Example: D is a tavern owner, who serves too much liquor to X, knowing that X arrived alone by car. D also does not object when X gets out his car keys and leaves. If X drunkenly runs over P, a court will probably hold that Xs conduct in negligently (drunkenly) driving, although intervening, was sufficiently foreseeable that it should not absolve D of liability.) EX: Derdiarian v. Felix Contracting Corp. - Dickens epileptic driver - lost consciousness while driving and hit the plaintiff, a subcontractor at a construction site. The plaintiff sued Dickens and the construction company, the latter on the theory that they negligently failed to maintain a safe work site.

The plaintiff prevailed at trial. The defendant appealed, claiming that the accident was unforeseeable and thus that they should bear no liability. Rule: The defendant will not be held liable if the intervening act of the third party is not a foreseeable consequence of the defendants negligence. Analysis: The court finds that it is not unforeseeable that negligently failing to secure a work site near a road might result in a driver entering the site and hurting someone. The court says that the question was properly left up to a jury. Example: Watson v. Kentucky P was injured when a third party lit a match and ignited gasoline vapor causing an explosion. The gasoline vapor was the result of the derailing of a rail car owned by D * If the third party inadvertently lit the match, then the negligence of the Defendant is a proximate cause of the Plaintiffs injury because it is reasonably foreseeable that someone might light a cigar on the street. In this instance the explosion could not have occurred without the gasoline first having been spilled.* If the act of the third party was intentional and done with the purpose of lighting the gasoline, then the Defendant is not liable because the Defendant could not have foreseen that someone would maliciously light a match with the purpose of causing an explosion. Because such an act is not reasonably foreseeable, the Defendant had no way to guard against it. 2. Criminally or intentionally tortious conduct: A third persons criminal conduct, or intentionally tortious acts, may also be so foreseeable that they will not be superseding. But in general, the court is more likely to find the act superseding if it is criminal or intentionally tortious than where it is merely negligent. Keys left in car, theft and the criminal get in the accident. Liability? Depends TYPICAL FACT PATTERNS: 1. Med Mal: D breaches Duty causes injury and then a dr makes things worse thru med mal that IS considered foreseeable as a matter of law. The compounding of injury as a med screw up in foreseeable. 2. Intervening Neg Rescue: D runs red light, breaks P leg. Good Sam tries to help, ends up breaking Ps shoulder trying to get him to safety. The D will be liable for both injuries. Foreseeable that a rescuer could come along and make things worse. 3. Intervening Reaction or protection forces: D runs red light, breaks P leg. P is at cross light w lots of other ppl. The other ppl freak out bc car is in intersection. They react stampede one person steps on the face of P who is on the ground. Leaves a perm scare. D is liable for disfigurement. 4. A subsequent disease or accident: D runs red light, breaks P leg. P gets cast/crutches. P loses his balance with new crutches, falls and breaks his arm. D is liable for broken arm. Theory is that if you leave someone in a weakened and debilitated condition its your fault and its foreseeable. In most other scenarios look at breach and ask why you characterize it as negligence in the first place. What is it about this behavior that makes me angry as D what am I worried about? If what happened is what you were worried about happening you have a foreseeable type of harm. EX: 3 ppl go to lunch. A order shrimp cocktail and its gone bad makes her sick. This is a breach bc reasonable people know that if you leave shrimp unrefrigerated it makes you sick. A goes to the bathroom. B goes to the ladies room to find her. In the process she slips in vomit, and breaks her arm. B sues the restaurant. Breach of duty was serving A rotten shrimp. Indirect cause. Did not happen immediately. Does bad shrimp foreseeable break arms? We are a little surprised. Therefore not w in the scope of the restaurants responsibility. Therefore their breach was the cause, but not the prox cause and therefore theyre not responsible.

D. Responses to defendants actions: Where the third partys intervention is a "normal" response to the defendants act, that response will generally not be considered superseding. This is true even if the response was not all that foreseeable. 1. Escape: For instance, if in response to the danger created by D, P or someone else attempts to escape that danger, the attempted escape will not be a superseding cause so long as it was not completely irrational or

bizarre. (Example: D, driving negligently, sideswipes Ps car on the highway. P panics, thrusts the wheel to the right, and slams into a railing. Even though most drivers in Ps position might not have reacted in such an extreme or unhelpful manner, Ps response is not sufficiently bizarre to constitute a superseding cause.) Fuller v. Preis - Following a collision with the defendant, the decedent suffered head injuries causing multiple serious epileptic seizures. After several months he committed suicide. Rule: An intentional intervening act like suicide does not sever the causal relationship with the primary tortfeasor, when it is shown that the decedent was incapable of resisting the impulse to kill themselves as a result of the tortfeasor 2. Rescue: Similarly, if Ds negligence creates a danger which causes some third person to attempt a rescue, this rescue will normally not be an intervening cause, unless it is performed in a grossly careless manner. D may be liable to the person being rescued (even if part or all of his injuries are due to the rescuers ordinary negligence), or to the rescuer. Rescue Doctrine- allows an injured rescuer to sue the party which caused the danger requiring the rescue in the first place 1. if you caused accident have to help 2. once you start you cant stop Requirements for achieving rescuer status: 1-Did Ds negligence place victim in imminent peril (actual or apparent)? 2-Did P reasonably conclude that victim was in peril? 3-Did P use reasonable care to rescue victim? Usually this is a question for the jury. If Rescuer meets all 3 D is liable to rescuer if rescuer is injured. POLICY- we want ppl to rescue Firefighters Rule: bars suits by professional rescuers who are injured by hazards of their profession If its not a foreseeable consequence of their duties, then they can still recover

Public Policy impact on Prox Cause


Kelly v. Gwinnell Hold: A host who serves liquor to an adult social guest, knowing both that the guest is intoxicated and that he will thereafter be operating a motor vehicle, is liable for injuries inflicted on a 3rd party as a result of the negligent operation of a motor vehicle by the adult guest when such a negligence is caused by intoxication EXPAND RULE: used to just apply to minors/commercial establishments. Bars - ABC laws - are liable Dissent: Unrealistic Burden. Different from commercial bar. Social host Majority: Public Policy (avoid drunk driving accidents) trumps imposition. Courts see themselves as in line w aggressive anti-drunk driving legislature. Not a prox cause reason *NJ Statue later cuts down this Rule. When a person reaches a certain BAC anybody in contact w them should have known - social host is not jointly and severally liable - only would pay a portion *NJ is alone in this in most other states for social hosts - there is no duty *DD precluded from liability. POLICY want to encourage ppl to be DDs Enright v. Eli Lilly Can't recover from drugs ingested by your gma, which hurt your mom in utero, which in turn gave you cerebral palsy Public Policy: Dont want to discourage drug companies from making drugs. Here - the harm could go on for generations. Cut off recovery for those who were ACTUALLY harmed. Gma and Mom could sue. Gatekeeping concerns

DAMAGES
I. PERSONAL INJURY DAMAGES GENERALLY Eggshell Skull: Once D has committed the other 3 elements of the tort - breached duty and is the cause - D is liable for all damages that result. Even if they are surprisingly great in scope/greater than expected. You take ur P as you find him. If ur P is weak or frail you got to pay. A. Actual injury required: In any action based on negligence, the existence of actual injury is required. Unlike intentional tort actions, nominal damages may not be awarded. 1. Physical injury required: Furthermore, P must usually show that he suffered some kind of physical harm. (Example: P may not recover where he sustained only mental harm, with no physical symptoms.) 2. Elements of damages: But once physical harm has been proven, a variety of damages may be recovered by P. These include: a. Direct loss: The value of any direct loss of bodily functions. (Example: $100,000 for the loss of a leg/disfigurements) b. Economic loss: Out-of-pocket economic losses stemming from the injury. (Examples: Medical expenses, lost earnings, household attendant.) c. Pain and suffering: Pain and suffering damages. d. Hedonistic damages: Damages for loss of the ability to enjoy ones previous life. (Example: Compensation for loss of the ability to walk/loss of enjoyment of life, even if loss of that ability has no economic consequences.) B. Consciousness required: Courts are split about whether P must be conscious of the loss in order to be able to recover damages. Some states (e.g., New York) do not allow hedonistic damages where P is in a coma. C. Future damages: P brings only one action for a particular accident, and recovers in that action not only for past damages, but also for likely future damages. 1. Present value: When P is recovering future values, courts generally instruct the jury to award P only the "present value" of these losses. P can argue for future inflation - Some Js adjust for future inflation 2. Periodic payments: Some states now allow D to force P to accept periodic payments in certain situations. These payments generally terminate upon Ps death. (Example: In New York medical malpractice cases, where the judgment is for more than $250,000, D may pay the judgment by purchasing an annuity for P, which will terminate on Ps death.) D. maximum recovery rule: Anderson v. Sears, Roebuck & Co. - A motion for remittitur is a request for the court to lower the damages levied against D. If granted, P may generally either accept the remittitur or request a new trial. * The maximum recovery rule is intended to protect the role of the jury as finder of facts. Judges must determine only the maximum award that the jury could reasonably have reached, based on all of the evidence as to each element of damages. Under the maximum recovery rule, a court will overrule a jurys determination of damages only if the damages exceed the maximum amount the jury could reasonably find based upon all of the evidence. * In the present case, five elements of damages are applicable: (i) past physical and mental pain; (ii) future physical and mental pain; (iii) future medical expenses; (iv) loss of earning capacity - (Have Life Expectancy Tables) permanent disability and (have to show Permanent loss ) (v) disfigurement. EX2: Richardson v. Chapman Because the determination of damages is a question for the trier of fact, the court will only issue a remittitur if the damages shock the judicial conscious (excessive based on the evidence) E. Tax: Any recovery or settlement for personal injuries is free of federal income tax. F. PER-DIEM argument: legal strategy break physical/mental suffering into hours/days. Gives appearance of mathematical precision juries like it bc damages are quantifiable G. The collateral source rule: At common law, P is entitled to recover her out-of-pocket expenses, even if P was reimbursed for these losses by some third party. This is known as the "collateral source rule." Montgomery Ward & Co. Inc. v. Anderson The collateral source rule prevents the introduction of evidence of payments received by an injured party from sources collateral to the wrongdoer - D should not get the benefit (Example: P has hospital bills of $100,000.

A health insurance policy owned by P pays every dime of this. When P sues D, and establishes liability, P may recover the whole $100,000 even though in a sense she has collected twice.). **Gratuitous benefits (spouse nurses u back to health) also covered 4 Exceptions: 1. Ps lying her condition brought on by another cause/illness she never took time off work she paid all out of pocket compelled to return to work early to forego additional med care 1. Statutory modifications: Nearly half the states have modified the common law collateral source rule in one way or another. 2. Subrogation: Where the common law rule remains in effect, P may not get a windfall after all. An insurance company that makes payments to P will normally be subrogated to Ps tort rights. That is, it is the insurance company, not P, who will actually collect any judgment from D up to the amount of the payments made by the insurer. H. Loss of Consortium: I. Rule of Avoidable consequences/Mitigation: P has a "duty to mitigate." That is, P cannot recover for any harm which, by exercise of reasonable care, he could have avoided. In particular, P cannot recover for any harm which would have been avoided had P sought adequate medical care. EX: ZImmerman v. Ausland - torn knee cartilage - In order to recover for permanent injuries, a plaintiff has a duty to mitigate those damages by submitting to treatment that would cure the damages if a reasonable person would do so under the same circumstances 1. Seat belt defense: In some states, failure to use a seat belt may deprive P of recovery under the duty to mitigate if D can show that P would not have been seriously injured had P worn a seat belt, D may escape liability for the avoidable injuries. J. Legislative Control of Amounts Recovered. CAPS/limits on Damages = tort reform think Med Mal

Property damage:
Chattel: IF completely destroyed or converted then you get the entire value. If damaged you get the difference . If deprived of its use get damages that value the use of which P has been deprived *If have a purely personal value (family heirloom) then may recover the value to the owner * Can recover for pets (but not emotional distress) *Consequential Damages (cost of recovering horses/lost profits while locked out of business) Property: Markets value of the property if sold on the open market at time and place of the wrong

PUNITIVE DAMAGES
A. Punitive damages generally: Generally not available - Punitive damages can be awarded to penalize a defendant whose conduct is particularly outrageous. Its an additional sum of $ over and above the compensation for the harm suffered, awarded for the purpose of punishing the D, Deterring D not to do it again, and deterring others. 1. Negligence cases: In cases of negligence (as opposed to intentional torts), punitive damages are usually awarded only where Ds conduct was "reckless" or "willful and wanton." EX: Cheatham v. Pohle Punitive damages = deterrent/punishment States have broad discretion, P's dont have a RIGHT to punitive damages. In Indiana P gets 25% and state gets 75%. Any amount of punitive damages given to the plaintiff is a "creation of state law." a. Product liability suits: Punitive damages are also frequently awarded in product liability suits, if P shows that D knew its product was defective, or recklessly disregarded the risk of a defect. 2. Constitutional limits: The U.S. Constitution places some but not severe limits on the award of punitive damages. a. Due process: A defendant might be able to show that a particular punitive damages award violated its Fourteenth Amendment due process rights. If a jury is given unlimited discretion in deciding whether and in what amount to award punitive damages, this may violate due process. However, the mere fact

that the punitive damages are large relative to the compensatory damages will not by itself lead to a due process violation. EX. State Farm Mutual Automobile Ins. Co. v. Campbell Rule: The Due Process Clause of the Fourteenth Amendment prohibits the imposition of grossly excessive or arbitrary punishments on a tortfeasor; the reason is that elementary notions of fairness enshrined in constitutional jurisprudence dictate that a person receive fair notice not only of the conduct that will subject him to punishment, but also of the severity of the penalty that a state may impose Gore: 4-1 Ratio Here: 145-1 - not reasonable Court does not want a bright line rule - particularly egregious conduct may justify a higher ratio Utah court give a 9-1 ratio (basically as far as they could go w out risking remand by SC) SC sets outer limit of what is constitutionally appropriate - thats probably 9-1. b. Exxon Shipping Co. v. Baker SC reasoned that although punitive damages were warranted, they may not exceed what Exxon already paid to compensate victims for economic losses, which was about US$500 million. 1 to 1 ratio between punitive and compensatory damages was "a fair upper limit" in maritime cases that involved recklessness, compared to the lower liability of negligence or the higher liability of intentional conduct. SC wants damages to be predictable.

DEFENSES IN NEGLIGENCE ACTIONS


I. CONTRIBUTORY NEGLIGENCE
A. General rule: At common law, the doctrine of contributory negligence applies. The doctrine provides that a plaintiff who is negligent, and whose negligence contributes proximately to his injuries, is totally barred from recovery

(even if 5% neg) still in 5 Js. )


(Example: P, while crossing the street, fails to pay attention. D, traveling at a high rate of speed while drunk, hits and kills P. Had P behaved carefully, he would have been able to get out of the way. Even though Ds negligence is much greater than Ps, P will be totally barred from recovery because of his contributory negligence, if the doctrine applies.) B. Standard of care: The plaintiff is held to the same standard of care as the defendant (i.e., the care of a "reasonable person under like circumstances"). C. Proximate cause: The contributory negligence defense only applies where Ps negligence contributes proximately to his injuries. The same test for "proximate causation" is used as where Ds liability is being evaluated. (Example: On the facts of the above example, suppose that D was traveling so fast that even had P been careful, D would still have struck P. P will not be barred by contributory negligence, because his negligence was not a "but for" cause, and thus not a proximate cause, of Ps injuries.) D. Claims against which defense not usable: Since the contributory negligence defense is based on general negligence principles, it may be used as a bar only to a claim that is itself based on negligence. 1. Intentional torts: the defense may not be used (he should have ducked when I punched) 2. Willful and wanton: Similarly, if Ps conduct is found to have been "willful and wanton" or "reckless," the contributory negligence defense will not be allowed. (But if Ds negligence is merely "gross," contributory negligence usually will be allowed.) The idea is that the defense does not apply where D disregards a conscious risk. 3. Negligence per se : Contributory negligence can usually be asserted as a defense even to Ds "negligence per se," i.e., his negligence based on a statutory violation. (But if the statute was enacted solely for the purpose of protecting a class of which P is a member, contributory negligence usually may not be asserted as a defense.) E. Last clear chance: The doctrine of "last clear chance" acts as a limit on the contributory negligence defense. If, just before the accident, D had an opportunity to prevent the harm, and P did not have such an opportunity, the existence of this opportunity (this last clear chance) wipes out the effect of Ps contributory negligence. (Example: P crosses the street without looking. D, who is traveling faster than the speed limit, discovers Ps plight shortly before the collision. D

tries to hit the brake, but negligently hits the accelerator instead. P never spotted Ds car at all. Ds discovery of the danger gave him a last clear chance to avoid the accident, which D failed to take advantage of. This last clear chance wipes out the effect of Ps contributory negligence, and P may recover against D.) Example2: Davies v. Mann P had an ass grazing on the side of a public highway Ds wagon came down the path at a smartish pace and knocked down the ass, killing it. RULE: If the defendant had an opportunity to avoid the accident after the plaintiff no longer had such an opportunity, and the defendant improperly did not avoid the accident, he is liable. Court were uncomfortable totally baring Ps claim so carved out this exception.

II. COMPARATIVE NEGLIGENCE


A D is always entitled to show that the P did not e did not exercise proper care (reasonable care under the circumstances/statutory standard (jaywalking not protecting ur own safety) for her own safety. A. Definition: A "comparative negligence" system rejects the all-or-nothing approach of contributory negligence. It instead attempts to divide liability between P and D in proportion to their relative degrees of fault. P is not barred from recovery by his contributory negligence, but his recovery is reduced by a proportion equal to the ratio between his own negligence and the total negligence contributing to the accident. (Example: P suffers damages of $100,000. A jury finds that P was 30% negligent and D was 70% negligent. P will recover, under a comparative negligence system, $70,000 $100,000 minus 30% of $100,000.) Discretionary act by jury. 1. Commonly adopted: 46 states have adopted some form of comparative negligence B. "Pure" versus "50%" systems: Only 13 states have adopted "pure" comparative negligence. Pure Modified- The rest completely bar P if his negligence is (depending on the state) "as great" as Ds, or "greater" than Ds. C. Multiple parties: Where there are multiple defendants, comparative negligence is harder to apply: 1. All parties before court: If all defendants are joined in the same lawsuit, the solution is simple: only the negligence due directly to P is deducted from his recovery. (Example: Taking all negligence by all parties, P is 20% negligent, D1 is 50% negligent, and D2 is 30% negligent. P will recover 80% of his damages.) 2. Not all parties before court: If not all defendants are before the court, hard questions arise concerning jointand-several liability. The issue is whether the defendant(s) before the court, who is/are found to be only partly responsible for Ps loss, must pay for the whole loss aside from that caused by Ps own fault. (Example: Ps accident is caused by the negligence of D and X. P sues D, but cant find or sue X. The jury finds that P was 20% responsible; D, 30% responsible; and X, 50% responsible. Ps damages total $1 million. It is not clear whether P can collect the full $800,000 from D. Under traditional "joint and several liability" rules, P would be able to collect this full $800,000.) a. Various approaches: States have four or five different approaches to this problem. Some states have completely abolished the doctrine of joint-and-several liability in comparative negligence cases (so that P could recover only $300,000 on the above example). An important emerging approach is the "allocative" approach, by which P and the Ds share the burden of an absent or insolvent defendant, in proportion to their own fault. (Example: On the facts of the above example, P would bear 2/5 and D 3/5 of the burden of Xs absence. So P would recover $600,000 total.) D. Last clear chance: Courts are split about whether the doctrine of last clear chance should survive in a comparative negligence jurisdiction. Doesnt make much sense. E. Extreme misconduct by D: If Ds conduct is not merely negligent, but "willful and wanton" or "reckless," most states nonetheless will reduce Ps damages. 1. Intentional tort: But if Ds tort is intentional, most comparative negligence statutes will not apply. F. Seat belt defense: The "seat belt defense" is increasingly accepted in comparative negligence jurisdictions. In this defense, D argues that Ps injuries from a car accident could have been reduced or entirely avoided had P worn a seat belt; Ps damages should therefore be reduced. 1. Contributory negligence jurisdictions: In most contributory negligence jurisdictions, courts refuse to allow the seat belt defense at all. That is, Ps failure to wear a seat belt does not count against his recovery in most courts. 2. Comparative negligence jurisdictions: But in states that have comparative negligence, the seat belt defense is more successful. There are various approaches: (1) D is liable only for those injuries that would have occurred even had P worn a seat belt; (2) D is liable for all injuries, with a reduction made equal to the percentage of Ps fault; and (3) D is liable for all injuries, but Ps fault reduces his recovery for those injuries that would have been avoided.

a. Effect of statute: Thirty-two states have mandatory seat belt use statutes. But the majority of these either prohibit the seat belt defense completely or make the defense almost valueless by allowing only a small reduction of damages.

III. ASSUMPTION OF RISK an affirmative def.


A. Definition: A plaintiff is said to have (1)assumed the risk of certain harm if she has (2)voluntarily consented to take her chances that harm will occur. Where such an assumption is shown, the plaintiff is, at common law, completely barred from recovery B. Express assumption: If P explicitly agrees with D, in advance of any harm, that P will not hold D liable for certain harm, P is said to have "expressly" assumed the risk of that harm. (Example: P wants to go bungee jumping at Ds amusement park. P signs a release given to him by D in which P agrees to "assume all risk of injury" that may result from the bungee jumping. If P is injured, he will not be able to sue D, because he has expressly assumed the risk. 1. Public policy against assumption: But even Ps express assumption of the risk will not bar P from recovery if there is a public policy against the assumption of the risk involved. a. Bargaining power: For instance, if Ds position as a unique provider of a certain service gives him greater bargaining power than P, and D uses this power to force P into a waiver of liability, the court is likely to find that public policy prohibits use of the assumption of risk doctrine. (Example: D is a public utility or common carrier, whom P must patronize because of Ds monopoly. Even if P expressly assumes the risk, this will probably not bar recovery.) (EXAMPLE2: Seigneur v. National Fitness Institute, Inc entered into a participation agreement with defendant health club. The participation agreement contained an exculpatory clause which purported to release defendant from all liability for injuries caused by defendant's negligence. Following plaintiff wife's injury at defendant's club, plaintiffs sued defendant. The exculpatory clause expressed a clear intent to release defendant from liability from all acts of negligence, and defendant did not provide an essential public service such that an exculpatory clause would be "patently offensive" to the citizens of Maryland. b. Intentional or willful misconduct: Public policy usually prohibits a waiver of liability for Ds willful and wanton or "gross" negligence, and for Ds intentionally tortious conduct. c. Health care: Courts almost never allow P to expressly assume the risk of harm with respect to medical services. (Example: Even if P signs a contract with D, her doctor, saying, "I agree not to sue you for malpractice if anything goes wrong with my operation," no court will enforce this.)

C. Implied assumption of risk: Even if P never makes an actual agreement with D whereby P assumes the
risk, P may be held to have assumed certain risks by her conduct. Here, the assumption of risk is said to be "implied." 1. Two requirements: For D to establish implied assumption, he must show that Ps actions demonstrated that she: (1) knew of the risk in question; and (2) voluntarily consented to bear that risk herself Via conduct Example: D owns a baseball team. D posts big signs at the gates warning of the danger of foul balls. P has attended many games, and in each game buys a seat right behind home plate, a place where she and all other fans know many foul balls are hit. If P is hit by a foul ball, she will not be able to recover against D even if D negligently failed to screen the home plate area. This is because P knew of the risk in question, and voluntarily consented to bear that risk. 2. Knowledge of risk: The requirement that P be shown to have known about the risk is strictly construed. For instance, the risk must be one which was actually known to P, not merely one which "ought to have been" known to her. Example: Rush v. Commercial Realty Co. P was injured when she fell through the floor of a detached privy. Plaintiff won a suit against the Defendant landlord, Assumption of the risk requires actual knowledge of a particular risk, appreciation of its magnitude and voluntarily encountering the risk. P had no other alternative 3. Voluntary assumption: The requirement that P consented voluntarily is also strictly construed. a. Duress: For instance, there is no assumption of the risk if Ds conduct left P with no reasonable choice but to encounter a known danger. (Example: P rents a room in a boarding house from D. She has to use a common bathroom at the end of a hallway. After the lease starts, a hole in the floor leading to the bathroom develops, and D negligently fails to fix it. P knows about the hole, but nonetheless steps in it while going to the bathroom. P will not be barred from recovery by an implied assumption of risk, because Ds conduct left P with no reasonable alternative but to walk down the hallway to get to the bathroom.)

b. Choice not created by D: Where it is not Ds fault that P has no reasonable choice except to expose herself to the risk, the defense will apply. (Example: P is injured and needs immediate medical help. He asks D who had nothing to do with the injury to drive him to the hospital, knowing that Ds car has bad brakes. P assumes the risk of injury due to an accident caused by the bad brakes, because Ps dilemma is not the result of Ds wrongdoing.) 4. Distinguished from contributory negligence: Often, Ps assumption of risk will also constitute contributory negligence. (Example: P voluntarily, but unreasonably, decides to take her chances as to a certain risk.) Blackburn v. Dorta - Florida 1977 - Three cases where consolidated by the Supreme Court of Florida to determine if the doctrine of implied assumption of the risk was viable as an absolute bar to recovery subsequent to adoption of comparative negligence. RULE: The concept of comparative negligence eliminates the need for the affirmative defense of implied assumption of the risk. Several categories of assumption of the risk have developed, including (i) distinctions between express and implied; (ii) between primary and secondary and (iii) between strict and implied (landlord/tenant) These Labels are not that meaningful - all were doing is assumption of the risk analysis - The legal inquiry is the same under a comp negligence test no need for implied assumption of the risk analysis Once a J has comp neg - IAofRisk does not add very much

IV. STATUTE OF LIMITATIONS


When does it start?? D has burden to plead SofL
A. Discovery of injury: If P does not discover his injury until long after Ds negligent act occurred, the statute of limitations may start to run at the time of the negligent act, or may instead not start to run until P discovered (or ought to have discovered) the injury. 1. Medical malpractice: In medical malpractice cases, statutes and case law today frequently apply the "time of discovery" rule. (Example: D performs an operation on P in 1970, and leaves a foreign object in Ps body. P discovers the problem in 1990, and sues immediately. The statute of limitations is six years on tort actions. Many, probably most, states today would allow P to sue, on the theory that the statute only started to run at the earliest time P knew or should have known that the object was left in his body.) 2. Fraud harm concealed statute tolls 3. Minor statute tolls till they reach age EXAMPLE2: Teeters v. Currey - D negligently performed a surgery on the Plaintiff, Teeters (Plaintiff) that was intended to prevent future pregnancy. Plaintiff did not discover the negligence until three years later when she gave birth. Plaintiff sued and Defendant answered claiming the statute of limitations as a bar to recovery. RULE: Under Tennessee law, the statute of limitations in medical malpractice actions begins to run when the plaintiff discovers, or reasonably should have discovered the negligent injury. - This cases take time to notice - its INSIDE ur body. You cant see it.

V. IMMUNITIES
A. Family immunity: The common law recognizes two immunities in the family relationship: between spouses, and between parent and child. 1. Husband and wife: At common law, inter-spousal immunity prevented suits by one spouse against the other for personal injury. (Examples: If W is injured while a passenger in a car driven negligently by H, W cannot sue H. If H intentionally strikes W, W cannot sue for battery.) (Freehe v. Freehe - abandoned the interspousal immunity rule, allowing spouses to bring suit against one another in tort.) a. Abolition: But over half the states have now completely abolished the inter-spousal immunity, even for personal injury suits. Other states have partially abolished it (e.g., not applicable for intentional torts, or not applicable for automobile accident suits). * Even if J still has spousal immunity DN apply for car accidents owe same duty as to any other passenger

2. Parent and child: At common law, there is an immunity that bars suit by a child against his parents or vice versa. Again, many (though not most) states have abolished this immunity, and others have limited it. * NO IMMUNITY if action is willful or wanton (sexual abuse) EX: Zellmer v. Zellmer Steparent En loco parentis? If you are ACTUALLY acting as the Parent/Stepparent - you are immune (provide financial support/discipline child) But just marrying the parent does not give you immunity * No duty to meet idealized standard want to keep parental autonomy B. Charitable immunity: Charitable organizations, as well as educational and religious ones, receive immunity at common law. 1. Abolished: But more than 30 states have now abolished charitable immunity. Others have cut back on the doctrine (e.g., abolished as to charitable hospitals, or abolished where there is liability insurance). (Abernathy v. Sisters of St. Mary's - The reasons for the common law rule of immunity for nongovernmental charitable organizations are no longer applicable - The doctrine of immunity for charitable organizations arose from the belief that it was better for the individual to suffer than for society to be deprived of the benefits of the charity. But today charities are big business, with most carrying insurance to pay for torts. To continue to allow immunity is not only unjust to the injured, but tends to breed neglect and irresponsibility.) * Waive immunity for intentional torts C. Governmental immunity: At common law, there is "sovereign immunity," preventing anyone from suing the government. 1. United States: Suits against the federal government are generally allowed today, under the Federal Tort Claims Act (FTCA). But the FTCA does not allow certain types of tort suits 2. State governments: State governments have traditionally had similar sovereign immunity. But many have abolished that immunity. 3. Local government immunity: Local government units (cities, school districts, public hospitals, etc.) have traditionally had sovereign immunity as well a. "Proprietary" functions: But even at common law, where a local government unit performs a "proprietary" function, there is no immunity. Proprietary functions are ones that have not been historically performed by government, and which are often engaged in by private corporations. (Examples: The running of hospitals, utilities, airports, etc., is generally proprietary, since these are revenue-producing activities; they can therefore be the subject of suit for personal injuries. Police departments, fire departments and school systems are not proprietary, and cannot be sued at common law.) b. Abolition: In any event, most states have abolished the general local government immunity, and some that have not done so allow suits where there is liability insurance.(Ex: Clarke v. Oregon Health Sciences Uni. BC Oregon govt imposed caps on damages - No immunity for individuals, yes immunity for hospital) C. PO-PO Riss v. NY Police authorities are not liable for failing to provide special protection to individuals threatened with harm. BUT - Unlike the present case, if police authorities undertake responsibilities to specific individuals and expose them to risks without adequate protection, tort actions are allowed. Evolution in the law = now we protect victims of domestic violence much more. Shifts in our thinking of what we expect from police. Police attitudes changed over last 2 decades EX: DeLong v. Erie County Having an emergency call system does not create a duty. However - Ms. DeLongs placing of the call and her further reliance when she was assured that help was on the way, does create a duty. When a municipality voluntarily assumes a duty and negligently performs the duty, the municipality may be held liable if its conduct somehow increased the risk to defendant. 4. Government officials: Courts often grant public officials tort immunity, even where their public employer could be sued.(Examples: Legislators and judges generally receive complete immunity, as long as their act is within the broad general scope of their duties. D. Employer Immunity: Workers Comp = immunity from tort liability

VICARIOUS LIABILITY
I. EMPLOYER-EMPLOYEE RELATIONSHIP
A. Respondeat superior doctrine: If an employee commits a tort during the "scope of his employment," his employer will be liable (jointly with the employee). Look to the person higher up want the deep pockets 1. Applies to all torts: The doctrine applies to all torts, including intentional ones and those in which strict liability exists, provided that the tort occurred during the scope of the employees employment. B. Who is an "employee": Respondeat superior is applied to all cases involving "employees," but not to most cases involving "independent contractors. " You must therefore distinguish between these two. 1. Distinction: The main idea is that an employee is one who works subject to the close control of the person who has hired him. An independent contractor, by contrast, although hired to produce a certain result, is not subject to the close control of the person doing the hiring. a. Physical details: The "control" required to make a person an employee rather than an independent contractor is usually held to be control over the physical details of the work, not just the general manner in which the work is turned out. (Example: Murrell v. Goertz A "newspaper boy" is likely to be an independent contractor, not an employee, because the newspaper usually controls only the general terms of employment such as the time by which the deliveries must take place not the physical details, such as whether the work should be done by bike or automobile. ) C. Scope of employment: Respondeat superior applies only if the employee was acting "within the scope of his employment" when the tort occurred. The tort is within the scope of employment if the tortfeasor was acting with an intent to further his employers business purpose, even if the means he chose were indirect, unwise or even forbidden. 1. Trips from home: Most courts hold that where an accident occurs where the employee is travelling from her home to work, she is not acting within the scope of her employment. If the employee is returning home after business, courts are divided. COMMUTE NOT under Scope of Employment BUT - if foreseeable - there is an exception to the "commings and goings" rule Drinking at work could be an example Just as breathing dangerous fumes as in this case EX: Bussard v. Minimed Irma Hernandez was an employee of defendant Minimed, Inc. Minimed sprayed pesticide overnight at their facility and Hernandez became ill the following day because of the spray and asked to leave work early. On her way home, Hernandez became dizzy and crashed into the car of plaintiff, Barbara Bussard, who was stopped at a red light. Bussard filed this action against Minimed for liability under the doctrine of respondeat superior. Rule: "'[F]oreseeability' as a test for respondeat superior merely means that in the context of the particular enterprise an employee's conduct is not so unusual or startling that it would seem unfair to include the loss resulting from it among other costs of the employer's business." 2. Frolic and detour: Even a detour or side-trip for personal purposes by an employee may be found within the scope of employment if the deviation was "reasonably foreseeable. " (Example: While D, a salesperson, is taking a two-hour trip to visit a business prospect, she makes a five-minute detour to buy a pack of cigarettes. If an accident occurred during the detour, this would probably be held to be "within the scope of employment," so that Ds employer would be liable. But a two-hour detour for personal business while on a one-day trip would probably not be within the scope of employment. ) Ex: O'Shea v. Welch O'Shea, sufferred injuries when the car he was driving was struck by a car driven by defendant, Welch. In his complaint, O'Shea alleged that Welch, an Osco employee, was acting within the scope of his employment at the time of the accident. Rule: "If an employee wholly abandons, even temporarily, the employer's business for personal reasons, the act is not within the scope of employment, and the employer is not liable under respondeat superior for the employee's conduct during that lapse." 6 Factors: 1. Employees Intent 2. Nature, time and place of deviation 3. The time consumed in the deviation 4. The work for which the employee was hired

5. The incidental acts reasonably expected by the employer 6. The freedom allowed the employee in performing job responsibilities 3. Forbidden acts: Even if the act done was expressly forbidden by the employer, it will be "within the scope of employment" if done in furtherance of the employment. (Example: D, a storekeeper, expressly orders his clerk never to load a gun while showing it to a customer. The clerk ignores this rule and loads the gun, the gun goes off and the customer is hurt. D will be liable because the loading, though forbidden, was done in furtherance of the employers business purposes, i.e., sale of guns. ) 4. Intentional torts: The fact that the tort is an intentional one does not relieve the employer of liability. (Example: X is a bill collector for D. X commits assault, battery and false imprisonment on P in attempting to collect a debt. D will be liable. ) a. Personal motives: But if the employee merely acts from personal motives, the employer will generally not be liable. (Example: Nurse at D hospital has always hated P because of a prior fight. While P is in the hospital, Nurse kills P. D will not be liable, because Nurse has obviously acted from personal motives, not in an attempt to further Ds business. )

II. INDEPENDENT CONTRACTORS


A. No general liability: One who hires an independent contractor is not generally liable for the torts of that person B. Exceptions: However, there are some important exceptions to the rule that an employer is not liable for the torts of his independent contractor: 1. Employers own liability: First, if the employer is herself negligent in her own dealings with the independent contractor, this can give rise to employer liability.(Example: D knows that the work to be done is hazardous if not done with special precautions. She chooses a contractor, X, who she should know will probably not do the work safely. X, performing the work negligently, injures P. D is liable for the consequences, because of her own negligence in selecting X. ) 2. Non-delegable duty: Second, there are some duties of care that are deemed so important that the person doing them will not be allowed to delegate them to anyone. (Examples: A city cannot delegate its duty to keep its streets in good repair; a business owner cannot delegate his duty to keep the premises safe for business visitors; a driver cannot delegate the duty to keep her brakes in good working order. ) EX: Maloney v. Rath - Duty to maintain one's breaks is a non-delegable duty (Its still mechanic fault - can file indemnity clause - Therefore distinguished from strict liability) - Just Dont want to leave P's to sort it all out. Rule: Generally when an activity carries the risk of death or serious bodily injury u cannot avoid liability for negligent maintenance by delegating the maintenance to an independent contractor 3. Inherently dangerous activities: Finally, one who employs an independent contractor will also be liable where the work is such that, unless special precautions are taken, there will be a high degree of danger to others. a. Unusual risk: This special rule of vicarious liability applies only to "peculiar risks," i.e., risks differing from commonly-encountered risks. (Example: D, a city, hires X, an independent contractor, to dig a sewer in the street. X leaves the trench unguarded without warning lights at night. D will be liable to P, who drives his car into the trench D knew or should have known that the work being done posed peculiar risks to motorists. )

III. JOINT ENTERPRISE


A. Generally: A "joint enterprise," where it exists, may subject each of the participants to vicarious liability for the others negligence. A joint enterprise is like a partnership, except that it is for a short and specific purpose (e.g., a trip). 1. Use in auto cases: The doctrine is used most often in auto accident cases. The negligence of the driver is imputed to the passenger (either to allow the occupant of a second car to recover against the passenger, or to prevent the passenger from recovering against the negligent driver of the other car under the doctrine of imputed contributory negligence). B. Requirements for joint enterprise: There are four requirements for a joint enterprise: (1) an agreement, express or implied, between the members; (2) a common purpose to be carried out by the members; (3) a common pecuniary interest in that purpose; and

(4) an equal right to a voice in the enterprise, i.e., an equal right of control. EX: Popejoy v. Steinle Mom and daughter get in car wreck on their way to buy a calf. Mom is killed. Brought suit against Dad/Husband under a joint venture theory. Can the trip to pick up Ms. Steinles daughters calf be characterized as a joint venture with her husband, so as to impose vicarious liability on the Defendants representatives? Rule: An interest in profit is necessary in order to impose vicarious liability on a party based on a joint venture theory.

IV. BAILMENT AND AUTO CONSENT STATUTES


A. Family purpose doctrine: The family purpose doctrine provides that a car owner who lets members of her household drive her car for their own personal use has done so in order to further a "family purpose," and is, therefore, vicariously liable. This doctrine is probably now accepted by less than half of American courts. (Example: Parent lets Child use the car. Child negligently smashes into P. In a state adopting the family purpose doctrine, Parent will be vicariously liable for Childs negligence. Notice that if the state has adopted an auto consent statute, as described below, the family purpose doctrine would not be necessary in this example. ) B. Consent statutes: Many, though not most, states have enacted "automobile consent statutes," which provide that the owner of a car is vicariously liable for any negligence committed by one using the car with the owners permission. 1. Scope of consent: If the use by the borrower goes clearly beyond the scope of that consent, there is no liability. (Example: O lets B use Os car, but forbids B to drive on the highway. B drives on the highway, and injures D. D probably cannot recover against O under the auto consent statute, because Bs use clearly exceeded the scope of Os consent. ) C. Non-statutory bailment: If there is no consent statute, the mere existence of a bailment does not make the bailor vicariously liable for the bailees negligence. Rental Companies are NOT liable except: 1. Negligence by bailor: But the bailor may be negligent herself in entrusting a potentially dangerous instrument to the bailee, where the bailor should know that the latter may use it unsafely. (Example: D, a car rental company, rents to X. D knows or should know that X is drunk. X immediately runs over P. P will be able to recover against D, not because of vicarious liability, but because D was directly negligent in entrusting the instrumentality to a drunk driver. )

V. IMPUTED CONTRIBUTORY NEGLIGENCE


A. Traditional rule: The common law recognized the doctrine of "imputed contributory negligence" in many three-party situations. That is, because of some relation between A and B, Bs suit against C might be defeated because of As contributory negligence, "imputed" to B. 1. Driver and passenger: For instance, traditionally a drivers negligence could be imputed to his passenger so as to prevent the passenger from recovering against the driver of another vehicle whose negligence contributed to a collision between the two cars. 2. Family: Similarly, a husbands negligence was frequently imputed to his wife, and vice versa. And a parents negligence was imputed to his child. (Example: Father fails to supervise Child. Child runs in the street and is hit by D. Traditionally, D could defend by saying, "Father negligently failed to supervise, and his negligence should be imputed to Child, thus giving me a complete defense. ") B. Modern rule: But in the vast majority of states today, contributory negligence will be imputed only if the relationship is one which would make the plaintiff vicariously liable if he were a defendant. (Example: Passenger rides in a car owned and driven by Driver. Driver collides with Trucker. Passenger is injured. Passenger sues Trucker. Trucker shows that Driver was negligent, and asserts that Drivers negligence should be imputed to Passenger. Traditionally, this argument would work. But today, most courts would reject it, because the driver-passenger relationship is not one in which the passenger would be vicariously liable for the drivers negligence if the passenger were being sued, unless a joint enterprise existed between them. ) 1. "Both ways" rule: Conversely, if the relationship is one which would give rise to vicarious liability, in most courts contributory negligence is still imputed. if negligence can be imputed, contributory negligence will also be imputed. Example: Company owns an expensive truck. Worker, who is an employee of Company, drives the truck on Companys business. While en route, he negligently collides with Driver, who is also negligent. Company sues Driver for damage to the truck. Since Company would be vicariously liable for any negligence by Worker in a suit

brought by Driver, Workers contributory negligence will be imputed to Company, thus barring Company from recovery from Driver in a contributory negligence jurisdiction. If the state follows comparative negligence, probably Companys recovery will be reduced by Workers comparative fault.

JOINT TORTFEASORS I. JOINT LIABILITY


A. Joint and several liability generally: If more than one person is a proximate cause of Ps harm, and the harm is indivisible, each defendant is liable for the entire harm. The liability is said to be "joint and several." (Example: D1 negligently scratches P. P goes to the hospital, where she is negligently treated by D2, a doctor, causing her to lose her arm. P can recover her entire damages from D1, or her entire damages from D2, though she cannot collect twice.) 1. Indivisible versus divisible harms: This rule of joint and several liability applies only where Ps harm is "indivisible," i.e., not capable of being apportioned between or among the defendants. If there is a rational basis for apportionment that is, for saying that some of the harm is the result of D1s act and the remainder is the result of D2s act then each will be responsible only for that directly-attributable harm. B. Rules on apportionment: 1. Action in concert: If the two defendants can be said to have acted in concert, each will be liable for injuries directly caused by the other. In other words, apportionment does not take place. (Example: Bierczynski v. Rogers D1 and D2 drag race. D1s car swerves and hits P. D2, even though his car was not part of the collision, is liable for the entire injuries caused by D1s collision, because D1 and D2 acted in concert - all may be liable for concurrent negligence, regardless of which of the individuals directly caused the injury) 2. Successive injuries: Courts often are able to apportion harm if the harms occurred in successive incidents, separated by substantial periods of time. (Example: D1, owner of a factory, pollutes Ps property from 19701980. D1 sells to D2, who pollutes Ps property from 1981-1990. The court will apportion the damage neither defendant will have to pay for damage done by the other.) a. Overlapping: It may be the case that D1 is jointly and severally liable for the harm caused by both her acts and D2s, but that D2 is liable only for his own. This is especially likely where D2s negligence is in response to D1s. (Example: D1 negligently breaks Ps arm. D2 negligently sets the arm, leading to gangrene and then amputation. D1 is liable for all harm, including the amputation. D2 is only liable for the amount by which his negligence worsened the condition that is, hes liable for the difference between a broken and amputated arm.) 3. Indivisible harms: Some harms are indivisible (making each co-defendant jointly and severally liable for the entire harm). a. Death or single injury: Thus the plaintiffs death or any single personal injury (e.g., a broken arm) is not divisible. b. Fires: Similarly, if Ps property is burned or otherwise destroyed, this will be an indivisible result. (Example: D1 and D2 each negligently contribute to the starting of a fire, which then destroys Ps house. There will be no apportionment, so D1 and D2 will each be liable for Ps full damages.) C. One satisfaction only: Even if D1 and D2 are jointly and severally liable, P is only entitled to a single satisfaction of her claim. (Example: P suffers harm of $1 million, for which the court holds D1 and D2 jointly and severally liable. If P recovers the full $1 million from D1, she may not recover anything from D2.)

ADD MORE ON JOINT TORTFEASORS

STRICT LIABILITY
I. ANIMALS
A. Trespassing animals: In most states, the owner of livestock or other animals is liable for property damage caused by them if they trespass on anothers land. This liability is "strict" even though the owner exercises utmost care to prevent the animals from escaping, he is liable if they do escape and trespass. B. Non-trespass liability: A person is also strictly liable for non-trespass damage done by any "dangerous animal" he keeps. 1. Wild animals: A person who keeps a "wild" animal is strictly liable for all damage done by it, as long as the damage results from a "dangerous propensity" that is typical of the species in question. (Example: D keeps a lion cub, which has never shown any violent tendencies. One day, the cub runs out on the street and attacks P. Even if D used all possible care to prevent the cub from escaping, he is liable for Ps injuries, because the cub is a wild animal and the damage resulted from a dangerous propensity typical of lions, that they can attack without warning. ) 2. Domestic animals: But injuries caused by a "domestic" animal such as a cat, dog, cow, pig, etc., do not give rise to strict liability unless the owner knows or has reason to know of the animals dangerous characteristics. (Example: Same facts as above example, except that the animal is a dog. If the dog has never attempted to bite anyone before, D is not liable. But if D knew or had reason to know that the dog sometimes attacks people, he would be liable. )

II. ABNORMALLY DANGEROUS ACTIVITIES


A. General rule: A person is strictly liable for any damage which occurs while he is conducting an "abnormally dangerous" activity. 1. Six factors: Courts consider six factors in determining whether an activity is "abnormally dangerous": (1) there is a high degree of risk of some harm to others; (2) the harm that results is likely to be serious; (3) the risk cannot be eliminated by the exercise of reasonable care; (4) the activity is not common; (5) the activity is not appropriate for the place where it is carried on; and (6) the danger outweighs the activitys value to the community. 3rd restatement advocates 2 element approach: 1. Whether the activity creates a foreseeable and highly significant risk of phys. harm even when all the actors exercise reasonable care (if you use a gun w reasonable care its not dangerous) 2. The activity is not a matter of common usage (guns are OK) 2. Requirement of unavoidable danger: Probably the single most important factor is that the activity be one which cannot be carried out safely, even with the exercise of reasonable care. Example: D, a construction contractor, carries out blasting operations with dynamite, to excavate a foundation. D uses utmost care. However, a piece of rock is thrown out of the site during an explosion, striking P, a pedestrian on the street. Blasting is an abnormally dangerous activity, in part because it cannot be conducted with guaranteed safety. Therefore, D will be strictly liable for the injury to P. B. Examples: Here are some types of activities that are generally held to be abnormally dangerous: 1. Nuclear reactor: Operation of a nuclear reactor 2. Explosives: The use or storage of explosives (see above example) 3. Crop dusting: The conducting of crop dusting or spraying 4. Airplane accidents: There usually is not strict liability in suits by passengers for airplane accidents. Therefore, in a suit by the estate of a passenger against the airline, the plaintiff must show negligence. (But most courts do impose strict liability for ground damage from airplane accidents. ) C. Rylands v. Fletcher - ENG 1865

Brief Fact Summary. Plaintiff sued in connection with the flooding of his mine. The trial court found in his favor. Defendant sought review. Synopsis of Rule of Law. A person who for his own purposes brings on his lands and collects and keeps there anything likely to do mischief if it escapes, must keep it in at his peril, and, if he does not do so, is prima facie answerable for all the damage which is the natural consequence of its escape.

Public Policy - building a reservoir in coal country - bad idea? Independent contractor - cant be held liable (engineer) POSSIBLE EXCUSE: o P causes harm o act of God o Natural use of the land

III. LIMITATIONS ON STRICT LIABILITY


A. Scope of risk: There is strict liability only for damage which results from the kind of risk that made the activity abnormally dangerous. (Example: D operates a truck carrying dynamite, and the truck strikes and kills P. P must show negligence. Transporting dynamite may be ultra-hazardous, but Ps death has not resulted from the kind of risk that made this activity abnormally dangerous. ) 1. Abnormally sensitive activity by plaintiff: A related rule is that D will not be liable for his abnormally dangerous activities if the harm would not have occurred except for the fact that P conducts an "abnormally sensitive" activity. (Example: Ds blasting operations frighten female mink owned by P; the mink kill their young in reaction to their fright. D is not strictly liable, because P was conducting an abnormally sensitive activity. [Foster v. Preston Mill Co.]) B. Contributory negligence no defense: Ordinary contributory negligence by P will usually not bar her from strict liability recovery. 1. Unreasonable assumption of risk: But assumption of risk is a defense to strict liability. Thus if P knowingly and voluntarily subjects herself to the danger, this will be a defense, whether P acted reasonably or unreasonably in doing so. (Example: P, an independent contractor, agrees to transport dynamite for D. P understands that dynamite can sometimes explode spontaneously. If such an accident occurs, P cannot recover from D in strict liability, because P has assumed the risk; this is true whether P acted reasonably or unreasonably. Example: Sandy v. Bushey - Sandy (Plaintiff), was injured when he was kicked by the Defendant, Busheys (Defendant), horse. Defendant knew that the horse had vicious propensities. Rule Contributory negligence is not a bar to recovery for an action in strict liability. BUT - can use CN as a defense if: 1. The injury was attributable not to the keeping of the animal, but to the injured party's unnecessarily 2. and voluntarily putting himself in a way to be hurt knowing the probable consequences of his act, so that he may fairly be deemed to have brought the injury upon himself a. AN ASSUMPTION OF RISK ARGUMENT - AND THAT WOULD BE A DEFENSE IN THIS CONTEXT C. Acts of God: Golden v. Amory Golden and others (Plaintiffs), claim that Amorys (Defendants), negligence in maintaining a dike caused them real estate damage after a hurricane resulted in a flood. RULE: Plaintiffs are not responsible under strict liability theories when the damage caused was an unanticipated act of God.

VII. PROCEDURE IN JURY TRIALS


A. Burden of proof: In a negligence case (as in almost all tort cases) P bears the "burden of proof." This is actually two distinct burdens: 1. Burden of production: First, P must come forward with some evidence that P was negligent, that P suffered an injury, that Ds negligence proximately caused the injury, etc. This burden is known as the "burden of production." This burden shifts from P to D, and perhaps back again during the trial.

2. Burden of persuasion: Second, P bears the "burden of persuasion." This means that as the case goes to the jury, P must convince the jury that it is more probable than not that his injuries are due to Ds negligence. B. Function of judge and jury 1. Judge decides law: The judge decides all questions of law. Most importantly, the judge decides whether reasonable people could differ as to what the facts of the case are; if they could not, he will direct a verdict. (Example: In a car accident case, if the judge decides that D drove so fast that no reasonable person could believe that D acted non-negligently, he will take this issue away from the jury by saying that they must find D negligent.) 2. Jury decides facts: The jury is the finder of the facts. In a negligence case (assuming that the judge does not direct a full or partial verdict), the jury decides: (a) what really happened; and (2) whether D breached his duty to P in a way that proximately caused Ps injuries. This means that it is the jury that usually decides whether Ds conduct satisfied the "reasonable person" standard.

Vous aimerez peut-être aussi